Human physiology Exam 1,3, and Final test bank

Réussis tes devoirs et examens dès maintenant avec Quizwiz!

A person's lowest metabolic rate A) occurs during sleep. B) occurs when they are younger. C) occurs during exercise. D) occurs after a meal. E) occurs when body temperature increases.

A

Anders is born with a mutation in the gene for CCK and he produces and secretes an inactive form of this hormone. Which of the following would be TRUE of Anders' digestion? A) A reduction in the release of pancreatic enzymes B) A reduction in the release of HCl C) An increase in the release of bile D) An increase in the amount of bile salts present in the bile E) None of these would occur

A

Andrew has his gall bladder removed after developing several large gall stones. After surgery, he cannot release large batches of bile into the duodenum, instead, bile is added slowly in small amounts. What nutrient group might Andrew have trouble digesting when consumed in larger quantities? A) Lipids B) Proteins C) Carbohydrates D) Nucleic acids E) None of these

A

As sperm travel from their site of production in the testes to the exterior, the sequence of ducts through which they will pass is A) rete testis; efferent ductules; vas deferens; ejaculatory duct; urethra. B) efferent ductules; rete testis; vas deferens; ejaculatory duct; urethra. C) rete testis; efferent ductules; vas deferens; urethra; ejaculatory duct. D) rete testis; afferent ductules; ejaculatory duct; vas deferens; urethra. E) rete testis; ejaculatory duct; efferent ductules; vas deferens; urethra.

A

Cholesterol is a A) precursor of steroid hormones. B) component of DNA. C) precursor of bile pigments. D) carbohydrate. E) vitamin.

A

Constriction of the ________ decreases hydrostatic pressure in ________. A) afferent arterioles; glomerular capillaries B) efferent arterioles; proximal convoluted tubules C) renal vein; peritubular capillaries D) efferent arterioles; glomerular capillaries E) efferent arterioles; Bowman's capsule

A

Drug X is a new drug to treat Alzheimer's Disease. This drug is able to cross the blood-brain barrier and circulate in the CSF. The drug is slightly acidic and at the recommended dose it decreases the CSF pH from 7.4 to 7.3. Will Drug X have any effect on respiration? A) It will cause an increase in respiratory rate. B) It will cause a decrease in respiratory rate. C) It will have no effect on respiratory rate, but will cause an increase in depth of breathing. D) It will have no effect on respiratory rate, but will cause the patient to take shallower breaths. E) It will not impact respiratory function.

A

During in utero (fetal) development of the ovary, which of these occurs? A) Cell division of primary oogonia results in the production of 2 to 4 million primary oocytes, which are present at birth. B) Cell division of primary oogonia, which all complete the first meiotic division before birth. C) Cell division of primary oogonia, which all complete both meiotic divisions before birth. D) The mitotic division of primary and secondary oocytes produces cells with 23 chromosomes each. E) A single primary oogonium remains dormant, which then undergoes rapid mitosis after birth.

A

During the absorptive state of metabolism, amino acids A) are used for protein synthesis in most body cells. B) are used for glycogen synthesis in the liver. C) may be converted to ketones in the liver. D) are burned as fuel in muscles, creating urea as a waste product. E) are mobilized from muscle and other tissues faster than they are taken up by those tissues.

A

Esperanza is a 9-year-old girl who was prescribed a drug that increases the secretion of all hypothalamic hormones. Regarding reproduction, which of the following is TRUE of Esperanza? A) Puberty will begin. B) Development of testes will occur. C) Maturation of ovaries will cease. D) Puberty will be delayed. E) None of these will occur.

A

Following a few months of noticeably high urine volume, Stephanie's doctor diagnosed her with diabetes insipidus and prescribed her desmopressin, which is a synthetic drug form of vasopressin. A few weeks later, Stephanie accidentally takes two doses instead of one. On that day, what is Stephanie likely to experience? A) An increase in the osmolarity of urine B) An increased urine volume C) A decrease in blood pressure D) A noticeable reduction in thirst and desire to drink water E) None of these would occur

A

Following crossing-over in meiosis I, which of the following is NOT true? A) Maternal and paternal chromatids will always orient to opposite poles of the cell, remaining segregated from one another. B) Random orientation of maternal and parental chromatids to either pole of the cell allows mixing of these in resultant gametes. C) It is extremely unlikely that all 23 maternal and paternal chromosomes will end up in the same gamete. D) A resulting gamete may have millions of possible different combinations of maternal and paternal chromosomes. E) Chromatid pairs lined up in the cell are separated so that half end up in each daughter cell when division occurs.

A

How would the kidneys respond to hypoventilation? A) Increasing secretion of H+ and increasing production of new HCO3- B) Increasing secretion of H+ and decreasing reabsorption of HCO3- C) Decreasing secretion of H+ and increasing production of new HCO3- D) Decreasing secretion of H+ and decreasing reabsorption of HCO3- E) Increasing excretion of CO2

A

If a patient with pulmonary disease began to hypoventilate, how would plasma levels of [H+], HCO3-, and PCO2 be changed, compared to normal? A) Increased [H+], increased PCO2, and increased [HCO3-] B) Increased [H+], increased PCO2, and decreased [HCO3-] C) Increased [H+], decreased PCO2, and decreased [HCO3-] D) Decreased [H+], increased PCO2, and decreased [HCO3-] E) Decreased [H+], decreased PCO2, and decreased [HCO3-]

A

If more calories are ingested in food than are expended by metabolism A) a person will gain weight. B) a person will lose weight. C) a person will stay the same weight because the excess energy is destroyed. D) whether weight is gained or lost depends upon whether the calories are in the form of proteins, carbohydrates, or fat.

A

If the amount of progesterone in the blood remained at the level seen in the middle of the luteal phase for 6 straight months, what would the effect be on menstruation? A) Menstruation would not occur. B) Menstruation would occur consistently for the entire 6 months. C) Menstruation would occur at the typical intervals, but would be heavier than before. D) Menstruation would occur at the typical intervals, but would be lighter than before. E) There would be no effect on menstruation.

A

If the nerves from the carotid and aortic bodies are cut, which of these would occur? A) A decrease in arterial PO2 will no longer increase ventilation. B) An increase in arterial PCO2 will no longer increase ventilation. C) An increase in arterial lactic acid will inhibit ventilation. D) An increase in arterial PCO2 will inhibit ventilation. E) A decrease in arterial PO2 will inhibit ventilation.

A

If you start at the plasma membrane of an oocyte in a mature follicle and move outward, which is the correct order in which you would encounter the three structures listed below? A) Zona pellucida; granulosa cells; theca cells B) Zona pellucida; theca cells; granulosa cells C) Granulosa cells; zona pellucida; theca cells D) Granulosa cells; theca cells; zona pellucida E) Theca cells; zona pellucida; granulosa cells

A

In Addison's disease, the adrenal cortices degenerate. Which will result if a patient with this disease undergoes prolonged fasting? A) Hypoglycemia B) Hyperglycemia C) Excessive lipolysis D) Decreased insulin sensitivity E) Excessive liver gluconeogenesis

A

In a hypothetical rare genetic disease in which the ribosomes in some cells of the anterior pituitary are malformed. Which hormone, if any, would you expect missing from a patient with this disease? A) Anti-diuretic hormone B) Thyroid hormone C) Progesterone D) Estrogen E) Oxytocin

A

In an effort to prolong the years of female reproduction a drug company produces a drug that prevents more than one follicle from maturing each menstrual cycle. Which of the following might be a side-effect of this drug? A) Lower circulating estrogen levels in the follicular phase B) Lower circulating progesterone levels in the follicular phase C) Lower circulating progesterone levels in the luteal phase D) Higher circulating estrogen levels in the luteal phase E) Lower libido

A

In the presence of aldosterone, which nephron region reabsorbs the greatest fraction of the filtered Na+? A) Proximal tubule B) Macula densa C) Descending limb of the loop of Henle D) Distal convoluted tubule E) Cortical collecting duct

A

In what segment of the nephron is the greatest fraction of filtered water reabsorbed? A) The proximal tubule B) The ascending limb of the loop of Henle C) The distal convoluted tubule D) The collecting ducts E) The descending limb of the loop of Henle

A

In which form is most of the oxygen carried in arterial blood? A) Bound to hemoglobin B) Dissolved in the plasma C) Dissolved in the cytosol of erythrocytes D) Converted to HCO3- E) Bound to myoglobin

A

It's final exam time and you're finding yourself extremely stressed out. If we examined your blood, which of the following would we be likely to find? A) Increased secretion of corticotropin from the anterior pituitary B) Increased secretion of corticotropin from the hypothalamus C) Increased secretion of adrenocorticotropic hormone (ACTH) from the hypothalamus D) Negative feedback to the adrenal cortex by adrenocorticotropic hormone (ACTH) E) Decreased secretion of corticotropin-releasing hormone (CRH) by the hypothalamus

A

Jim has just been diagnosed with testicular cancer. The tumor is composed of Sertoli cells and they have begun to produce excessive amounts of inhibin. Which of the following would be TRUE? A) FSH secretion would be selectively inhibited, but LH secretion would be normal. B) Spermatogenesis would be increased but testosterone secretion would be normal. C) Spermatogenesis would be increased because testosterone production would be above normal. D) Gonadotropin-releasing hormone levels and spermatogenesis would increase above normal. E) Both FSH and LH secretion would rise above normal levels.

A

Julia comes to see her doctor with a variety of symptoms. Her doctor eventually diagnoses her with gallstones. Which of the following was NOT one of Julia's symptoms? A) Lactose intolerance B) Abdominal pain C) Difficulty digesting high-fat meals D) Nutritional deficiencies E) Jaundice

A

Lactose intolerance A) is an inability to digest milk sugar. B) is most common in very young children. C) results from the inability to secrete insulin. D) results from removal of the gallbladder. E) is an inability to digest proteins.

A

Many forms of birth control prevent ovulation, although follicular maturation continues to occur. Which of the following would NOT occur in a woman on one of these forms of birth control? A) Completion of the first meiotic division of the oocyte B) The second meiotic division of the oocyte C) Expulsion of the thecal cells onto the ovarian surface D) All of these will still occur E) Expulsion of the corpus luteum from the ovary

A

Most of the reabsorption of filtered calcium is unregulated and occurs in the ________, but an additional amount can be reabsorbed in the ________, depending on the plasma concentration of parathyroid hormone. A) proximal convoluted tubule; distal convoluted tubule B) distal convoluted tubule; proximal convoluted tubule C) proximal convoluted tubule; descending loop of Henle D) distal convoluted tubule; medullary collecting ducts E) distal convoluted tubule; renal corpuscle

A

Parturition A) is the delivery of the infant and placenta. B) normally occurs between the 27th and 28th weeks of pregnancy. C) is an example of negative feedback. D) occurs within 24 to 48 hours of ovulation. E) is a pathological condition in which a pregnant woman retains too much fluid (edema), and it can lead to seizures.

A

Priya has been having trouble conceiving. She went through testing at a fertility clinic and found that everything seemed fine with gametogenesis and the uterine cycle, but that her cervical mucus seemed abnormally thick. The doctor suggested a supplement to help thin the mucus and allow sperm to get through the mucus to the uterus, but Priya can't remember which type of supplement it was. If the supplement were to provide benefit, it could work through increasing which of the following? A) Estradiol B) Progesterone C) Androgen D) Inhibin E) Drinking more water

A

Rebecca's younger sister just told her that she has begun menopause. Rebecca wonders if she has begun menopause as well. One of the symptoms to watch out for is A) the cessation of menstrual cycles. B) high plasma levels of estrogens. C) cessation of gonadotropin secretion. D) the cessation of ovulation while a baby is nursing. E) the pause in development of ovarian follicles occurring between the birth of a baby girl and when she reaches puberty.

A

Sally is a 10-year-old girl going through a growth spurt. Her blood levels of growth hormone are two-fold higher than at the same time last week. Which other hormones are likely to also be elevated? A) Insulin-like growth factor 1 B) Prolactin C) Cortisol D) ACTH E) Follicle-stimulating hormone

A

Sarah is a type 1 diabetic who forgot to bring insulin with her today. Which of the following will be TRUE of Sarah? A) Her muscle and adipose tissue cells will not be able to take up as much glucose from the bloodstream. B) Her brain cells will not be able to take up as much glucose from the bloodstream. C) Her muscle cells will not be able to take up as many amino acids from the bloodstream. D) Her liver will not breakdown glycogen. E) None of these will occur.

A

Sperm A) are stored in the vas deferens and the last part of the epididymis. B) travel from the seminiferous tubules to the epididymis by the beating action of their flagellae. C) travel through the vas deferens, then the ejaculatory duct, then the ureter during ejaculation. D) are expelled from the penis through the ureter. E) are no longer produced in individuals who have undergone a vasectomy.

A

Sydney had an infection 10 years ago that left scarring in her fallopian tubes. Due to this scarring, the diameter of the fallopian tubes has been reduced by 75%. What effects may Sydney experience? A) Oocytes may not reach the uterus. B) Endometrium may not mature as needed for implantation. C) During pregnancy the fetus may not mature to full size. D) The cervix may not dilate during birth. E) None of these would occur.

A

Talia is a busy store clerk on Black Friday. She ate breakfast at 7 am but was not able to take a lunch break because the store was too busy. Its now 3 pm and she's beginning to feel a bit odd. Which of the following does NOT describe Talia's physiological state? A) Hyperglycemia is occurring. B) Gluconeogenesis is occurring. C) Glucose-sparing reactions are occurring. D) Insulin secretion is lower than in the absorptive state. E) Glucagon secretion is higher than in the absorptive state.

A

The blastocyst implants in the uterus on about the ________ day of the 28-day menstrual cycle. At this time, the endometrium is ________ and progesterone levels are ________. A) 21st; thick and secretory; high B) 28th; thick and secretory; low C) 14th; thick and secretory; low D) 28th; thin and proliferating; high E) 21st; thin and proliferating; low

A

The major metabolic effects of glucagon include A) stimulating glycogenolysis and gluconeogenesis in the liver. B) stimulating triglyceride synthesis in adipose tissue. C) stimulating glucose uptake into skeletal muscle cells. D) stimulating the activity of glycogen synthase in the liver. E) inhibiting the enzyme glycogen phosphorylase in the liver.

A

The primary kind of motility in the stomach during digestion of a meal is A) peristalsis. B) segmentation. C) relaxation of the pyloric sphincter. D) mass movement. E) regurgitation.

A

The type of smooth muscle contraction occurring in the esophagus during swallowing is called A) peristalsis. B) mass movement. C) segmentation. D) distension. E) eccentric.

A

There is an E. coli outbreak in the town of Springfield when almost all the Springfield residents contracted food poisoning at a town hall meeting. E. coli causes severe diarrhea. The local Springfield pharmacy is ordering supplies to stock its shelves as its customers work their way through the outbreak. Which of the following would be helpful for the pharmacy to stock? A) Antacids to help the residents recover from acidosis B) Acidic foods such lemonade to help the residents recover from alkalosis C) Diuretic drugs to help the residents recover from high blood pressure D) Vitamin B12 supplements to help the residents recover from this specific deficiency E) All of these would be helpful

A

Tom is on a health kick and wants to make sure that he is always in balance in terms of total body water. Which of the following is important for him to keep in mind? A) He must ingest more water than is lost in the urine. B) He must ingest more water than is lost by all output pathways combined. C) He must ingest less water than is lost in the urine. D) The water filtered into Bowman's capsule must be 100% reabsorbed. E) The amount ingested plus that metabolically produced must equal the amount of water in the urine.

A

Tracey has been experiencing a fever and a feeling that she needs to urinate every few minutes, even though she has a very low urine volume. She goes to her doctor who tells her that she has a bacterial infection of her bladder. An ultrasound of her ureters indicates that there is no infection in the ureters. Where did the infection begin? A) Her urethra B) Her nephrons C) Her renal arteries D) Her renal pelvis E) None of these sites

A

Type II alveolar cells are among the last cells to mature during fetal development. In babies born prematurely, type II alveolar cells are often not ready to perform their function. Which of the following would be TRUE of these babies? A) They would be at risk of alveolar collapse due to too much surface tension in the alveoli. B) They would be at risk of bacterial infections in the lungs. C) They would be at risk of autoimmune diseases with lung complications. D) They would be more likely to have coughing fits. E) None of these would occur.

A

Type II alveolar cells are among the last cells to mature during fetal development. In babies born prematurely, type II alveolar cells are often not ready to perform their functions. Which of the following drugs would be most likely to help an infant with immature type II alveolar cells? A) Pulmonary surfactant B) A beta-adrenergic agonist C) A muscarinic agonist D) Histamine E) A beta-adrenergic antagonist

A

Under normal circumstances, which of the following would result from an increase in transpulmonary pressure? A) Inhalation/inspiration B) Exhalation/expiration C) A collapsed lung D) Pneumothorax E) Emphysema

A

Water and NaCl reabsorbed from the loop of Henle directly reenter which blood vessels? A) Vasa recta B) Afferent arterioles C) Efferent arterioles D) Cortical peritubular capillaries E) Collecting ducts

A

What is one function of the type II alveolar cells? A) Production of surfactant B) Secretion of mucus C) Phagocytizing bacteria and other foreign particles D) Make up the majority of the epithelial wall of the alveoli E) Lining the pleural space

A

What is the primary regulator of the magnitude of alveolar ventilation under normal circumstances? A) The H+ concentration in the brain extracellular fluid, which is monitored by central chemoreceptors B) The PO2 of the arterial blood, which is monitored by central chemoreceptors C) The PO2 of the arterial blood, which is monitored by peripheral chemoreceptors D) The H+ concentration in the arterial blood, which is monitored by central chemoreceptors E) Stretch receptors in the lung

A

What would be the effect of giving an agonist to the oxytocin receptor to a woman late in pregnancy? A) Onset of uterine contractions B) Inhibition of breast milk production C) Declining levels of estrogen D) Increase in maternal blood pressure E) Inhibition of cervical dilation

A

When blood Ca2+ levels fall below normal, in what ways do the kidneys help restore them toward normal? A) By increasing 1,25-dihydroxyvitamin D3 formation, decreasing tubular phosphate reabsorption, and increasing tubular Ca2+ reabsorption B) By increasing 1,25-dihydroxyvitamin D3 formation, increasing tubular phosphate reabsorption, and increasing tubular Ca2+ reabsorption C) By decreasing 1,25-dihydroxyvitamin D3 formation, increasing tubular phosphate reabsorption, and increasing tubular Ca2+ reabsorption D) Increasing 1,25-dihydroxyvitamin D3 formation and increasing secretion of parathyroid hormone E) By increasing renal secretion of parathyroid hormone and increasing bone resorption

A

When examining an endocrine cell from a new species of mammal under the microscope you find that it contains an abundance of rough endoplasmic reticulum, a large Golgi apparatus, and secretory granules. Which of the following is most likely to be true about these cells? A) It secretes a peptide/protein hormone. B) It secretes its product by diffusion through the lipid bilayer of the plasma membrane. C) It secretes by endocytosis. D) Its hormone product is synthesized from cholesterol. E) It secretes a hormone with a hydrophobic structure.

A

Which correctly describes an action of the hormone insulin? A) It increases the uptake and utilization of glucose by muscle and adipose tissue cells. B) It is required for the uptake and utilization of glucose by nervous tissue. C) It decreases the uptake of amino acids by muscle cells. D) It stimulates the breakdown of glycogen in the liver. E) It inhibits the action of lipoprotein lipase in the capillaries of adipose tissue.

A

Which correctly describes renin? A) It is secreted by juxtaglomerular cells in renal afferent arterioles. B) Its secretion is enhanced by high levels of Na+ in the macula densa. C) Its secretion is stimulated by elevated blood pressure in the renal afferent arterioles. D) It acts on the adrenal cortex to stimulate aldosterone secretion. E) It is secreted by the liver in response to low blood pressure.

A

Which could be an outcome in a woman who had lower than normal levels of the human placental lactogen during pregnancy? A) Lower fat content of breast milk after delivery B) Degeneration of the corpus luteum C) Weight loss by the mother D) Vasoconstriction and increased maternal blood pressure E) Premature uterine contractions

A

Which is NOT true about the actions of progesterone? A) It increases contractions and activity of the fallopian tube smooth muscle and cilia. B) It induces thick, sticky mucus in the cervix. C) It inhibits the milk-inducing effects of prolactin. D) It increases body temperature. E) It exerts negative feedback on the hypothalamus and anterior pituitary.

A

Which is TRUE about gastric motility? A) The basic electrical rhythm of the gastric smooth muscle (three depolarizations per minute) is the same regardless of whether or not food is present. B) Gastric contractions are strongest in the fundus of the stomach. C) The force of contraction is decreased by gastrin and increased by enterogastrones. D) When a wave of excitation reaches the pyloric sphincter, action potentials become inhibitory and the sphincter opens wide to allow chyme to exit. E) The term "slow waves" refer to gastric contractions, while "pacemaker" refers to gastric action potentials.

A

Which is TRUE regarding meiosis in a single germ cell? A) It results in 4 cells with 23 chromosomes each. B) It results in 2 cells with 46 chromosomes each. C) It results in 4 cells with 46 chromosomes each. D) It results in 4 cells with 46 pairs of chromosomes each. E) It results in 4 cells with 23 pairs of chromosomes each.

A

Which is TRUE regarding the secretion of HCl in the stomach? A) H+ is actively transported into the gastric lumen by ATPase pumps in the mucosal membrane. B) H+ is actively cotransported into the lumen by secondary active transporters coupled to K+. C) The pH of the blood leaving the area of the parietal cells is lower than the normal ECF value of 7.4. D) H+ diffuses passively from mucosal epithelial cells into the lumen of the stomach, following Cl-. E) Cl- and HCO3- are exchanged across the luminal membrane of epithelial cells via a passive transport protein.

A

Which is TRUE regarding the structure and synthesis of hormones? A) Steroid hormones are synthesized from cholesterol. B) Thyroid hormones are catecholamines. C) The hormones of the adrenal cortex have the same structure as the neurotransmitters of adrenergic neurons. D) Most peptide hormones require binding proteins for transport in the blood. E) Vasopressin is synthesized in the posterior pituitary.

A

Which is TRUE regarding the three phases of the control of HCl secretion? A) In the intestinal phase, an increase in osmolarity of the contents of the duodenum decreases HCl secretion. B) In the gastric phase, distension of the stomach decreases HCl secretion. C) In the cephalic phase, increased nutrient concentration in the duodenum stimulates HCl secretion. D) In the intestinal phase, increased peptide concentration in the stomach inhibits HCl secretion. E) In the gastric phase, decreased H+ concentration in the stomach inhibits HCl secretion.

A

Which is a function of LH? A) It stimulates androgen production by theca cells. B) It stimulates the growth of granulosa cells. C) It stimulates production of inhibin by theca cells. D) It stimulates estrogen production by theca cells. E) It inhibits GnRH secretion by the hypothalamus during ovulation.

A

Which is most likely a result of hypersecretion of growth hormone? A) Acromegaly B) Dwarfism C) Decreased plasma concentration of IGF-I D) Decreased secretion of somatostatin E) Increased secretion of growth hormone-releasing hormone (GHRH)

A

Which is normally TRUE about the intrapleural pressure? A) It is lower than alveolar pressure. B) It is between +5 and +10 mmHg above atmospheric pressure at functional residual capacity. C) It alternates between being less than, and greater than, atmospheric pressure. D) During a passive exhale, it increases to a value above atmospheric pressure. E) It is always the same as atmospheric pressure during a passive exhale.

A

Which of the following conditions would result in a systemic arterial PO2 lower than is typical of a healthy person at sea level? A) Traveling to high altitude B) Breathing 100% oxygen C) Having iron-deficiency anemia D) Breathing regular air in a hyperbaric chamber (higher that normal atmospheric pressure) E) Maintaining alveolar ventilation constant while decreasing metabolic rate

A

Which of the following contributes to a catabolic state? A) Increased plasma cortisol concentration B) Hypothyroidism C) Increased plasma androgen concentrations D) Increased plasma insulin concentration E) Decreased plasma glucagon concentration

A

Which of the following is NOT a function of the respiratory system? A) Net uptake of carbon dioxide from the air and removal of oxygen from the blood B) Regulation of blood H+ concentration C) Trapping of blood clots D) Phonation E) Defense against microbes

A

Which of the following is NOT true regarding ventilation-perfusion inequality? A) No inequality exists in normal lungs. B) It may result from increased alveolar dead space. C) It is increased from normal in patients with emphysema. D) It is caused by too little or too much blood flow relative to ventilation. E) It is caused by too little or too much ventilation relative to blood flow.

A

Which of the following is TRUE about how water is handled by the nephron? A) Water is filtered out of glomerular capillaries by bulk flow. B) Water is actively reabsorbed from the proximal tubule, and Na+ follows down its diffusion gradient. C) Water is actively secreted into the descending loop of Henle. D) The permeability of the ascending limb of the loop of Henle is modified by vasopressin. E) Vasopressin inserts pumps in the collecting duct membrane that move water against its concentration gradient.

A

Which of the following is an action of the sympathetic nervous system during stress? A) It increases the ability to respond to situations where physical activity is required. B) It increases blood flow to the skeletal muscles and viscera. C) It inhibits blood clotting. D) It decreases ventilation of the lungs to save oxygen. E) It increases glycogen synthesis in the liver.

A

Which of the following is inhibited by progesterone during pregnancy? A) The sensitivity of the myometrium to stimuli that promote contraction B) Plugging of the uterus by thickened cervical mucus C) Growth of the mammary glands D) Secretion of estrogen E) Regression of the corpus luteum

A

Which of the following is secreted by the chief cells of the gastric mucosa? A) Pepsinogen B) HCl C) Intrinsic factor D) Gastrin E) Pepsin

A

Which of the following is the most potent inhibitor (directly or indirectly) of gastric motility and emptying? A) Fat in the duodenum B) Acid in the stomach C) Carbohydrate in the stomach D) Gastrin E) Distension of the stomach

A

Which of the following statements about renal control of blood acid-base balance is TRUE? A) Increased metabolism of glutamine by renal tubular cells increases the plasma bicarbonate concentration. B) Excretion in the urine of hydrogen bound to phosphate buffers decreases plasma bicarbonate concentration. C) H+ that binds to filtered bicarbonate in the tubular fluid is excreted in the urine. D) When hypoventilation occurs at the lungs, the kidneys compensate by reducing glutamine metabolism. E) The kidneys compensate for a metabolic alkalosis by increasing CO2 production.

A

Which of the following statements concerning the process of glomerular filtration is correct? A) The hydrostatic pressure in Bowman's space opposes filtration. B) The glomerular filtration rate is limited by a transport maximum. C) All of the plasma that enters the glomerular capillaries is filtered. D) The osmotic force due to plasma proteins favors filtration. E) The hydrostatic pressure in glomerular capillaries opposes filtration.

A

Which of the following statements regarding Ca2+ homeostasis is TRUE? A) Parathyroid hormone directly stimulates Ca2+ reabsorption by the kidneys. B) Parathyroid hormone directly stimulates Ca2+ absorption from the GI tract. C) In the absence of parathyroid hormone, plasma Ca2+ levels would be abnormally low, resulting in the hyperpolarization of nerve and muscle membranes. D) When plasma Ca2+ increases above normal, the secretion of parathyroid hormone increases. E) Vitamin D decreases the renal tubular reabsorption of Ca2+.

A

Which of the following statements regarding hormonal regulation of gastrointestinal function is TRUE? A) An increase of H+ in the small intestine stimulates secretion of the hormone secretin, which in turn stimulates HCO3- secretion by the pancreas. B) The presence of fatty acids in the stomach stimulates secretion of CCK, which in turn stimulates enzyme secretion by cells in the antrum of the stomach. C) The presence of amino acids in the small intestine stimulates secretion of gastrin, which in turn stimulates HCl secretion by parietal cells. D) The presence of fatty acids in the small intestine stimulates the secretion of the hormone secretin, which causes contraction of the gallbladder. E) The hormone somatostatin stimulates the secretion of H+ into the lumen of the stomach.

A

Which of the following statements regarding hypoxia is correct? A) Exposure to high altitude is a form of hypoxic hypoxia. B) "Anemic hypoxia" refers to the condition of lower than normal arterial PO2. C) Carbon monoxide poisoning is an example of hypoxic hypoxia. D) Carbon monoxide poisoning is an example of ischemic hypoxia. E) Cyanide poisoning is an example of hypoxic hypoxia.

A

Which of the following statements regarding lipid digestion and absorption is TRUE? A) The presence of fatty acids in the duodenum stimulates the secretion of CCK, which in turn stimulates secretion of pancreatic enzymes. B) The presence of fatty acids in the duodenum stimulates the secretion of secretin, which in turn stimulates contraction of the gallbladder. C) The presence of fatty acids in the duodenum stimulates the secretion of gastrin, which in turn stimulates bile synthesis in the gallbladder. D) The presence of fatty acids in the duodenum stimulates emptying of the stomach. E) The presence of fatty acids in the stomach stimulates the secretion of secretin, which inhibits motility of the large intestine.

A

Which of the following statements regarding neural regulation of gastrointestinal function is TRUE? A) Networks of neurons in the wall of the GI tract innervate the wall's smooth muscle. B) Smooth muscle of the GI tract is innervated by both sympathetic and somatic motor nerves. C) If the autonomic nerves to the GI tract were cut, digestion and absorption of food could no longer take place. D) The nerve plexus of the GI tract exists in a single layer, just outside of the serosa. E) There are no afferent neuronal pathways from the GI tract to the central nervous system.

A

Which of the following would be most likely to cause metabolic alkalosis? A) Severe vomiting B) Severe diarrhea C) Hyperventilation D) Strenuous exercise E) Hypoventilation

A

Which of these describes secondary hypersecretion of hormone X? A) A growing tumor secretes hormone Y, which stimulates the gland that secretes hormone X. B) Cells of a growing tumor manufacture hormone X in unregulated fashion. C) Negative feedback from a tumor that hypersecretes hormone Z inhibits the gland that secretes hormone X. D) Hormone X is secreted by a growing tumor that is in the anterior pituitary gland. E) Hormone X is secreted in unregulated fashion by a tumor growing in a tissue that does not normally secrete hormone X.

A

Which results from insulin binding to its receptor in skeletal muscle plasma membranes? A) Glycogen synthase is stimulated. B) Glycogen phosphorylase is stimulated. C) Lipoprotein lipase is inhibited. D) Glucose transport into the cell is inhibited. E) Protein catabolism is stimulated.

A

Which stimulates vasopressin secretion? A) Increased plasma osmolarity B) Increased plasma volume C) Ingestion of alcohol D) Decreased aldosterone secretion E) Increased pressure in afferent arterioles

A

Which would be the effect of a blood clot that blocked the vessels of the hypothalamo-pituitary portal system in the infundibulum between the hypothalamus and the anterior pituitary gland? A) Secretion of adrenocorticotropic hormone (ACTH) would decrease and the adrenal cortex would atrophy B) Secretion of gonadotropins will decrease and the gonads will hypertrophy C) Secretion of corticotropin-releasing hormone (CRH) would decrease D) Secretion of prolactin would decrease E) Secretion of follicle-stimulating hormone (FSH) would increase

A

With regard to fat metabolism, insulin increases A) the activity of lipoprotein lipase, which leads to increased triglyceride synthesis. B) the export of fatty acids and monoglyceride from adipocytes. C) the activity of hormone-sensitive lipase, which synthesizes triglycerides. D) the activity of hormone-sensitive lipase, which catabolizes triglycerides.

A

With regard to reproduction, which is correct about SRY? A) It is a protein that is normally coded for by a gene that is present on the Y chromosome. B) It is a protein normally coded for by a gene that is present on the X chromosome. C) It is a gene that codes for testosterone. D) It is a protein that causes the degeneration of the Wolffian ducts. E) It is a gene found in both males and females.

A

Yolanda was born with a rare genetic disease in which the proximal tubules of her nephrons express half the number of aquaporins as is typical. What will be the result? A) Reduced reabsorption of water, although as much reabsorption of Na+ as is typical B) Reduced reabsorption of both water and Na+ in equal proportions C) Reduced reabsorption of all materials in the proximal tubules D) Reduced reabsorption of water, Na+, and glucose in the proximal tubules E) No change to reabsorption at this site

A

You are a new technician in an assisted-reproduction clinic. You've been asked to fill-in for your colleague who is sick today. Her job was to use ultrasound to assess sperm production in a male client who had previously undergone a vasectomy. Where will the sperm be found? A) The vas deferens and last part of the epididymis B) The seminiferous tubules C) The ureter D) None of these locations as this client no longer produces sperm

A

You are a pediatrician. You have a young patient who does not seem to be growing. You suspect that there might be stress in the home. You decide to perform a blood test for cortisol and expect to find A) high plasma cortisol levels have led to overall state of catabolism. B) high plasma cortisol levels have led to a decrease in available fuel in the bloodstream. C) high plasma cortisol levels have led to a decrease in blood calcium levels. D) high plasma cortisol levels have led to an increased skeletal muscle metabolic rate, there are no nutrients left for growth. E) None of these answers are correct.

A

You are a scientist for the Federal Drug Administration (FDA). An agricultural company wants to market a new pesticide they have developed for use on produce in the United States. This pesticide is hydrophilic, has a small molecular weight (it is slightly smaller than glucose), and it does not bind to any proteins. Which of the following is an appropriate hypothesis for how this drug will be found in the body? A) The pesticide will mostly be cleared in the feces as it will bind to cholesterol, which is added to the GI tract. B) The pesticide will remain in the blood for a long period as it is not filtered or secreted by the urinary tract. C) The pesticide will mostly be cleared in the urine as it will be filtered but not reabsorbed. D) The pesticide will be cleared through the skin as it will diffuse through the skin layers and evaporate from the surface.

A

You are volunteering at a weight-loss clinic. You're in charge of a station that makes individualized recommendations for increasing daily metabolic rate. Which of the following lifestyle changes can increase metabolic rate to the greatest extent? A) Increasing skeletal muscle activity B) Increasing mental activity C) Increasing dietary caloric intake D) Increasing environmental temperature E) Increasing blood glucose levels

A

You walk into your dorm room and find your roommate in the midst of anxiety-induced hyperventilation. You're unable to calm her down, so you call an ambulance. When she arrives at the ER, which of the following is her likely diagnosis? A) Respiratory alkalosis B) Respiratory acidosis C) Increased blood PCO2 D) Metabolic acidosis E) Metabolic alkalosis

A

A class of drugs that inhibit vasodilation would have what effect on male reproduction? A) Orgasm would occur more readily B) Erection may be inhibited C) Increased rate of sperm production may occur D) Increased volume of semen may be produced E) None of these effects would occur

B

A decrease in metabolism without a similar, compensatory decrease in alveolar ventilation will have what effect on the systemic pH? A) pH will decrease. B) pH will increase. C) No change to pH is expected in this circumstance. D) It is impossible to predict the effect on pH without first understanding why metabolism decreased.

B

A group of students has declared a hunger strike to protest a perception of low-quality dorm food. Jose is one of the protesting students and he has not eaten for three days. A blood test reveals that there are high levels of ketones in Jose's blood. These ketones A) are a waste product of amino acid metabolism that the kidneys must excrete. B) can be used by the brain as an energy source. C) are a by-product of anaerobic metabolism. D) are converted to fatty acids and released into the blood. E) are used to synthesize glucose by gluconeogenesis.

B

A person who has hypothyroidism would be expected to A) have an increased basal metabolic rate. B) have reduced tolerance for cold temperatures. C) lose body weight. D) be restless and irritable. E) have symptoms similar to over-activation of the sympathetic nervous system.

B

A person without a gallbladder A) cannot secrete bile. B) cannot store bile. C) will have no difficulty digesting a large, fat-rich meal. D) cannot digest fats. E) cannot store lipase.

B

Andres has a parathyroid hormone deficiency. He first approached the doctor because he was experiencing ________ and the early blood tests found that he had ________. A) decreased muscular excitability; low plasma calcium levels B) increased muscular excitability; low plasma calcium levels C) decreased muscular excitability; high plasma calcium levels D) increased muscular excitability; high plasma calcium levels E) frequent bones breaks; high plasma calcium levels

B

As a result of a new medication, Jessica's adrenal glands have increased their production of androgen. On the medication's label, Jessica reads about the potential side effects. Which of these is likely to be included in the list of potential side effects? A) Onset of spermatogenesis in her ovaries B) Increased levels of LH secretion C) Atrophy of her skeletal muscles D) Increase of subcutaneous fat deposition E) New or increased growth of facial hair

B

Aziz goes to the doctor and has a blood cholesterol test done. The results show that his total cholesterol is slightly high, his HDLs are very high, and his LDLs are low. Aziz is alarmed by the numbers because his father had atherosclerosis. Aziz's doctor gives him more information. Which of the following statements is the doctor likely to make? A) It is preferable to have a high concentration of low-density lipoproteins (LDLs) in the plasma because LDLs operate to remove cholesterol from the peripheral tissues and carry it to the liver, which secretes it into the small intestine. B) It is preferable to have a high concentration of high-density lipoproteins (HDLs) in the plasma because HDLs operate to remove cholesterol from the peripheral tissues and carry it to the liver, which secretes it into the small intestine. C) It is preferable to have equal concentrations of LDL and HDL. D) Neither LDL nor HDL are involved in cholesterol regulation. E) Plasma cholesterol concentration has no relationship to atherosclerosis.

B

Carbonic anhydrase catalyzes a reaction that combines which of the following? A) H2O and O2 B) H2O and CO2 C) H2O and CO D) H+ and HCO3- E) CO2 and O2

B

Dietary amino acids in a meal that are in excess of the body's need to make protein are A) converted to glucose in the liver. B) converted to fatty acids in the liver. C) excreted without being absorbed. D) converted to fatty acids in adipose cells. E) burned for fuel by cells of the nervous system.

B

During a long illness, Terrell has not been eating much. As a result, his plasma Ca2+ levels decrease. Which hormone would we expect to see in higher levels than usual in his blood? A) Thyroxine B) Parathyroid hormone C) Calcitonin D) Growth hormone E) Insulin

B

During a physical examination, Joe learns that his resting tidal volume is 550 mL; his average resting respiratory rate is 15 breaths per minute; his total lung capacity is 6,000 mL; and his anatomic dead space is 150 mL. Joe's resting alveolar ventilation is A) 72.0 L/min. B) 6.0 L/min. C) 4.2 L/min. D) 1.8 L/min. E) 0.5 L/min.

B

During a run, your gastrocnemius (calf) muscle has a temperature of 38.5° C, while your biceps brachii (upper arm) muscle has a temperature of 37.5° C. Which of the following is TRUE of the red blood cells as they pass by these muscles? A) The hemoglobin molecules will have the same affinity for oxygen at both locations. B) The hemoglobin molecules will have a higher affinity for oxygen as they pass by the biceps brachii compared to the gastrocnemius. C) The hemoglobin molecules will have a higher affinity for oxygen as they pass by the gastrocnemius compared to the biceps brachii. D) The hemoglobin molecules may denature as they pass by the gastrocnemius.

B

During a yoga class you take a deep breath, then relax and let the breath out slowly. During that exhalation, which would NOT be true? A) Alveolar pressure is greater than atmospheric pressure. B) Intrapleural pressure is greater than alveolar pressure. C) Intrapleural pressure becomes less negative. D) The diaphragm relaxes. E) Lung volume decreases.

B

During exercise A) blood glucose levels fall dramatically. B) epinephrine and glucagon stimulate glycogenolysis in liver cells. C) decreased blood glucose and increased epinephrine stimulate insulin secretion. D) epinephrine stimulates glucagon secretion and inhibits skeletal muscle uptake of glucose. E) cortisol and growth hormone secretion are decreased.

B

During the postabsorptive phase of metabolism A) glycogen in muscle cells is broken down to glucose, which is released into the blood. B) liver glycogen is broken down to glucose, which is released into the blood. C) triacylglycerols in adipose tissue are broken down to glucose by lipolysis. D) the liver exports lactate to the muscles, which use it for fuel. E) glycerol and fatty acids are rapidly assembled into triglycerides in adipose tissue.

B

Excess absorbed glucose is initially converted to triacylglycerides and very low-density lipoprotein by A) adipose cells. B) liver cells. C) intestinal mucosa cells. D) muscle cells. E) lymphoid cells.

B

Following a few months of noticeably high urine volume, Stephanie's doctor diagnosed her with diabetes insipidus. Stephanie's doctor prescribed her desmopressin, which is a synthetic drug form of which of these hormones? A) ACTH B) Vasopressin C) Atrial natriuretic factor D) Angiotensin II E) Insulin

B

Following an intense workout your clothes are drenched in sweat. You estimate you lost at least 0.5 L of sweat. Your (similarly sized) friend skipped her workout to donate blood and you found out she donated 0.5 L of plasma. After these activities, which of you will have a greater increase in vasopressin secretion and why? A) You will have a greater vasopressin secretion because sweat is hyperosmotic to plasma. B) You will have a greater vasopressin secretion because sweat is hypoosmotic to plasma. C) Your friend will have a greater vasopressin secretion because sweat is hypoosmotic to plasma. D) Your friend will have a greater vasopressin secretion because sweat is isosmotic to plasma. E) Your friend will have a greater vasopressin secretion because sweat is hyperosmotic to plasma.

B

Gluconeogenesis is stimulated by decreased levels of A) epinephrine. B) insulin. C) glucocorticoids. D) growth hormone. E) glucagon.

B

If the adrenal glands were removed from a patient, his plasma cortisol levels would ________, secretion of CRH by the ________ would ________, and secretion of ACTH by the ________ would ________. A) increase; hypothalamus; decrease; anterior pituitary gland; decrease B) decrease; hypothalamus; increase; adenohypophysis; increase C) decrease; anterior pituitary gland; increase; hypothalamus; increase D) increase; hypothalamus; increase; adenohypophysis; increase E) decrease; adenohypophysis; increase; anterior pituitary gland; increase

B

If the pituitary gland is removed from a human subject, which of the following is likely to occur? A) The adrenal gland will hypertrophy to increase cortisol production. B) The adrenal gland will atrophy and plasma cortisol levels will be reduced. C) The hypothalamus will secrete less CRH. D) The subject will remain euthyroid. E) There will be increased negative feedback on growth hormone by insulin-like growth factor 1 (IGF-1).

B

Implantation of a blastocyst A) normally occurs within the oviduct (fallopian tube). B) normally begins around day 21 of the typical menstrual cycle. C) occurs within 24 hours of fertilization of the oocyte. D) takes place at a time in the typical menstrual cycle when estrogen concentration in the blood is higher than progesterone concentration in the blood. E) requires the presence of high concentrations of follicle-stimulating hormone (FSH).

B

In subjects with type 2 (non-insulin-dependent) diabetes mellitus, which of these occurs? A) Pancreatic beta cell responses to increased plasma glucose concentration are normal. B) Target tissues respond poorly to insulin. C) Hypoglycemia is typically the first symptom. D) Severe ketoacidosis is common.

B

In the condition diabetes mellitus, why does glucose appear in the urine? A) The plasma concentration of glucose becomes so high that it diffuses from peritubular capillaries into the proximal tubule, down its concentration gradient. B) The filtered load of glucose becomes greater than the tubular maximum for its reabsorption. C) Without the hormone insulin, glucose cannot enter proximal tubule epithelial cells. D) The rate of tubular secretion of glucose becomes greater than the sum of glucose filtration and reabsorption. E) Without insulin, the glomerular filtration barrier becomes extremely leaky to glucose, which is not normally filterable.

B

In the ovarian cycle, the dominant follicle A) undergoes atresia. B) continues to grow beyond day 7 of the cycle, while the other developing follicles degenerate. C) is selected on day 1 of the cycle, and no other follicles develop beyond that day. D) is selected to continue growing at about the middle of the luteal phase. E) ruptures during ovulation on day 21 of the cycle.

B

Julie is 8 weeks pregnant. She takes a drug that is an antagonist to the progesterone receptor. What will happen? A) Ovulation will be disrupted. B) The endometrium and conceptus will be shed. C) Ovulation will occur. D) Follicles will begin to mature. E) Non-dominant follicles undergo atresia.

B

Kathryn is pregnant and very eager to breast-feed after the baby is born. Her friend gives her a supplement to encourage prolactin secretion and Kathryn starts to take it during pregnancy. The supplement works and Kathryn's blood level of prolactin increases. She does not begin lactating during pregnancy because A) the breast ducts do not develop until after parturition. B) the high levels of estrogen and progesterone in maternal plasma inhibit the effects of prolactin. C) maternal plasma levels of prolactin are low during pregnancy. D) high levels of androgens inhibit milk synthesis by the breasts. E) the low levels of circulating estrogen and progesterone during pregnancy inhibit milk synthesis by the breasts.

B

Liz is 5 weeks pregnant. During an ultrasound of her ovaries, which of the following can be seen? A) No follicles beyond primordial follicle stage B) One large corpus luteum and no other follicles beyond primordial stage C) Several follicles in primordial, primary, and preantral stages and one corpus luteum D) No follicles E) 10-25 mature follicles

B

On day 1 of the menstrual cycle, FSH levels will be ________. The follicle that will become dominant is in the ________ stage of maturation. Progesterone levels in blood are ________. A) rising; primordial; low B) rising; preantral to antral; low C) rising; preantral to antral; high D) falling; expanding; low E) falling; atretic; high

B

On day 14 of a typical 28-day menstrual cycle, LH concentration in the blood will be ________. The endometrium will be ________. Estrogen concentration in the blood is ________. A) level; sloughing off; low B) at or near a peak; near the end of the proliferative phase; relatively high C) low but rising; in the secretory phase; high D) at or near a peak; in the secretory phase; low E) level; starting the proliferative phase; level but high

B

Shedding of the endometrial lining occurs A) because ovulation disrupts the growing follicle. B) as a result of a fall in plasma levels of estradiol and progesterone. C) a day after the LH surge. D) because of an increase in the level of FSH. E) because the nondominant follicles undergo atresia.

B

The exact causes of inflammatory bowel diseases continue to be explored, but it is relatively clear that A) it is equally common among people of all ages and racial groups within the population. B) it is likely to result from a combination of environmental and genetic factors. C) individuals suffering from its effects are largely experiencing the consequences of the absence of immune mechanisms in the gastrointestinal tract. D) it can be easily diagnosed since its symptoms are always very specific and the affected areas are always limited to a very focused area within the small intestine. E) the cause is allergy to a single particular food, and patients get immediate relief by simply not eating that food.

B

The primary kind of motility in the small intestine during digestion of a meal is A) peristalsis. B) segmentation. C) receptive relaxation. D) migrating motility complex. E) mass movement.

B

The transition between the ________ and the ________ phases of the uterine cycle coincides with ________. A) follicular; luteal; a rise in basal body temperature B) proliferative; secretory; ovulation C) menstrual; proliferative; ovulation D) secretory; proliferative; selection of the dominant follicle E) proliferative; secretory; menstruation

B

The volume of air flowing into the alveoli during inhalation/inspiration is increased when there is an increase in which of these? A) Airway resistance B) The pressure gradient from the atmosphere to the alveoli C) The pressure in the intrapleural space D) The curvature of the diaphragm E) The volume of air in the intrapleural space

B

To synthesize thyroid hormones, iodine is attached to A) cholesterol. B) the amino acid tyrosine. C) the amino acid tryptophan. D) acetyl coenzyme A. E) the amino acid phenylalanine.

B

What structure secretes the enzyme that breaks down starch to smaller carbohydrates? A) The esophagus B) Pancreatic exocrine cells C) Pancreatic duct cells D) The liver E) The large intestine

B

When lying down, a subject's respiratory rate is 12 breaths per minute, his anatomical dead space is 150 mL, and his minute ventilation is 7 L/min. Which of these is closest to his alveolar ventilation in liters per minute? A) 6.0 B) 5.2 C) 4.2 D) 3.0 E) 0.583

B

When plasma glucose concentration increases, the secretion of insulin is ________ and the secretion of glucagon is ________. An increase in sympathetic activity will cause ________ secretion of insulin and ________ secretion of glucagon. A) decreased; increased; increased; decreased B) increased; decreased; decreased; increased C) increased; decreased; increased; decreased D) increased; increased; decreased; increased E) decreased; decreased; increased; unchanged

B

Which accurately describes the median eminence of the hypothalamus? A) It is the site of synthesis of adrenocorticotropic hormone (ACTH). B) It is a site where neurohormones are released into blood vessels. C) It is the site where vasopressin is released into blood vessels. D) It is the stalk connecting the hypothalamus to the posterior pituitary. E) It is the main site where thyroid-stimulating hormone first enters the bloodstream.

B

Which correctly lists a function of estrogens in females and testosterone in males? A) They prevent the pubertal growth spurt. B) They are responsible for epiphyseal plate closure. C) They stimulate the secretion of gonadotropin-releasing hormone (GnRH). D) They stimulate the secretion of luteinizing hormone (LH). E) They stimulate the reabsorption of bone.

B

Which is NOT an effect of parathyroid hormone? A) Increases the bone-degrading activity of osteoclasts B) Decreases reabsorption of calcium by the kidneys C) Promotes vitamin D synthesis, leading to increased intestinal absorption of calcium D) Increases plasma [Ca2+] E) Decreases reabsorption of phosphate by the kidneys

B

Which is TRUE about the hormone vasopressin (also known as antidiuretic hormone, ADH)? A) It is a peptide hormone released from the adrenal gland. B) It triggers insertion of aquaporins into the apical membranes of collecting duct cells. C) It promotes the excretion of more water in the urine. D) It stimulates the excretion of K+ in the urine. E) Its main function is to trigger the secretion of aldosterone.

B

Which is TRUE about the pleural sac? A) It is continuous with the atmosphere and protects the lungs from infection. B) It is a closed sac surrounding each lung that contains only a tiny volume of lubricating fluid. C) It is a closed sac surrounding each lung and contains a large volume of surfactant. D) It is a closed, air-filled space surrounding both lungs that has a negative pressure compared to atmospheric pressure. E) It is bounded by pleural membranes that are very thin, which allows for abundant gas exchange.

B

Which is TRUE regarding renal tubular reabsorption? A) Reabsorption of Na+ from the proximal tubule occurs as a result of water reabsorption. B) Reabsorption of glucose saturates at a maximum transport rate. C) Urea reabsorption cannot occur at any point along the nephron. D) Toxic substances are removed from the body by reabsorption from peritubular capillaries into the proximal tubule. E) Reabsorption of Na+ only occurs from nephron regions that come after the descending limb of the loop of Henle.

B

Which is a TRUE statement about implantation of a blastocyst? A) When it occurs outside the uterus, it is called a myopic pregnancy. B) It normally occurs midway through the luteal phase. C) It is a result of digestion of myometrial cells by enzymes in the trophoblast. D) It occurs at a time in the cycle when plasma estrogen concentration is higher than progesterone concentration. E) It occurs at the same time as, and is caused by, the LH surge.

B

Which is a result of elevated thyroid hormone levels? A) An increase in triglyceride storage B) Increased Na+/K+ ATPase activity C) Decreased body temperature D) Mental lethargy

B

Which is characteristic of a person chronically suffering from hypoxic hypoxia? A) Higher-than-normal alveolar PO2 B) Higher-than-normal hematocrit C) Higher-than-normal arterial PO2 D) Lower-than-normal lactic acid production E) Lower-than-normal blood flow through tissues

B

Which is most likely a characteristic of cells that secrete steroid hormones? A) They store large amounts of hormone. B) They are characterized by abundant smooth endoplasmic reticulum and few secretory granules. C) They contain large numbers of secretory granules. D) They are found in the anterior pituitary gland. E) They are found in the medulla of the adrenal gland.

B

Which occurs during the secretion of hydrochloric acid by gastric epithelial cells? A) There is a decrease in the pH of blood in the hepatic portal circulation. B) Bicarbonate ions are secreted into the hepatic portal circulation. C) Chloride ions are pumped from the cytosol of the gastric epithelial cells into the hepatic portal circulation blood. D) Both H+ and K+ are actively pumped from the epithelial cell cytosol into the lumen of the stomach. E) Vesicles containing H+/K+ -ATP-ase proteins are endocytosed into vesicles within the gastric epithelial cells.

B

Which of the following acts to decrease blood glucose levels? A) Glucagon B) Insulin C) Growth hormone D) Epinephrine E) Cortisol

B

Which of the following are chemoreceptors? A) Hair cells in the cochlea B) Receptors in the aortic and carotid bodies C) Baroreceptors in the aortic and carotid arch D) Receptors that mediate the Hering-Breuer reflex

B

Which of the following breakfasts would likely remain in the stomach longest? A) Toast, orange juice, and coffee B) Fried eggs, bacon, and hash browns C) A bowl of cereal with skim milk D) A boiled egg, toast, and juice E) The type of meal has no effect on the rate of gastric emptying.

B

Which of the following does NOT correctly describe kidney function? A) They contribute significantly to long-term regulation of arterial blood pressure by maintaining the proper plasma volume. B) They produce urine of a constant composition at all times, in order to maintain homeostasis of extracellular fluid. C) They excrete metabolic waste products. D) They assist in maintaining proper acid-base balance in the body. E) They secrete hormones.

B

Which of the following drugs, administered as an aerosol spray, would be most likely to help a patient during an asthmatic attack? A) Pulmonary surfactant B) A β2-adrenergic agonist C) A muscarinic agonist D) Histamine E) A β2-adrenergic antagonist

B

Which of the following hormones is NOT secreted by the placenta during pregnancy? A) Human placental lactogen B) Androgens C) Estradiol D) Progesterone E) Inhibin

B

Which of the following hormones is NOT usually secreted at higher levels in response to stress? A) Aldosterone B) Gonadotropins C) Vasopressin D) Antidiuretic hormone (ADH) E) Beta-endorphin

B

Which of the following is FALSE regarding erection of the penis? A) It is a spinal reflex. B) It is a result of sympathetic stimulation of vascular smooth muscle in the erectile tissue of the penis. C) It can be triggered by stimulation of tactile receptors in the penis. D) It is a result of arteriolar vasodilation and venous constriction of blood vessels in the penis. E) It can be inhibited by input to the autonomic nerves from higher brain centers.

B

Which of the following is NOT characteristic of inflammatory bowel disease (IBD)? A) Bleeding, edema, and ulceration anywhere along the gastrointestinal tract B) Pain relief when changing to a diet higher in fiber C) Perforations in the mucosa and intestinal wall, leading to infection by bacteria, which are normally present and benign D) Inflammation and thickening of the bowel wall to the point of preventing the usual passage of feces E) Pain in the lower right abdomen frequently mistaken for appendicitis

B

Which of the following is TRUE regarding pituitary growth hormone (GH)? A) The liver produces a factor that mediates the metabolic actions of GH. B) GH stimulates insulin-like growth factor-1 (IGF-1) production by the liver and by many other cells. C) GH increases the sensitivity of tissues to the action of insulin. D) GH exerts negative feedback on its own production by inhibiting the hypothalamic secretion of somatostatin. E) IGF-1 stimulates the secretion of GH by anterior pituitary gland cells.

B

Which of the following is a cause of asthma? A) Loss of alveoli B) Inflammation of the bronchioles C) Elevation of intrapleural pressure to equal atmospheric pressure D) Environmental chemicals that stimulate β2-adrenergic receptors E) Lack of pulmonary surfactant

B

Which of the following is a hormonal change that occurs near the end of a healthy pregnancy? A) Plasma progesterone levels decrease while the levels of estrogen continue to increase. B) Estrogens stimulate connexin synthesis in the myometrium. C) Plasma estrogen levels decrease while the levels of progesterone continue to increase. D) Estrogen blocks the synthesis of oxytocin receptors on uterine smooth muscle. E) There is a surge in the LH concentration.

B

Which of the following is least likely to be filtered into Bowman's capsule in a normal, healthy person? A) Glucose B) Plasma protein C) Sodium D) Urea E) Bicarbonate ion

B

Which of the following leads to an increase in the ratio of LDL cholesterol to HDL cholesterol? A) Estrogens B) Cigarette smoking C) Exercise D) Weight reduction E) All of these increase the LDL:HDL ratio.

B

Which of the following male accessory reproductive organs secretes prostaglandins? A) Prostate gland B) Seminal vesicles C) Epididymis D) Bulbourethral glands E) Vas deferens

B

Which of the following most accurately describes the renal transport of Na+? A) Na+ is actively transported in all segments of the tubule. B) Primary active transport of Na+ allows for secondary active transport of glucose and H+ in the proximal tubule. C) Most of the Na+ transport occurs in the distal convoluted tubule and collecting ducts. D) Na+ is actively secreted into the nephron lumen by cells in the cortical collecting ducts. E) Na+ is actively transported across the luminal membrane of proximal tubule cells in exchange for K+, by Na+/K+ ATPase pumps.

B

Which of the following organs is the main site of aldosterone secretion? A) Kidneys B) Adrenal glands C) Systemic and pulmonary blood vessels D) Liver E) The atria of the heart

B

Which of the following processes is NOT associated with gastrointestinal function? A) Digestion B) Filtration C) Secretion D) Motility E) Absorption

B

Which of the following statements about hormones is correct? A) Anterior pituitary hormones are synthesized in the hypothalamus. B) All hormones secreted by the anterior and posterior pituitary glands are peptides. C) IGF-1 stimulates growth by increasing growth hormone secretion through positive feedback. D) All of the hypophysiotropic hormones are peptides. E) Only steroid hormones can regulate the secretion of steroid hormones.

B

Which of the following statements about oxytocin is TRUE? A) Target cells of oxytocin have receptors for the hormone in their nucleus. B) Oxytocin is synthesized in the hypothalamus. C) Oxytocin is secreted by the anterior pituitary gland. D) Oxytocin's main function is to increase the rate of respiration. E) Oxytocin keeps uterine smooth muscle from contracting, so it prolongs pregnancy.

B

Which of the following statements about the transport of the absorbed products of fat digestion is TRUE? A) Free fatty acids and monoglycerides are transported to the liver via the hepatic portal vein. B) Products of fat digestion first go to the lymph system, and then to the veins leading to the heart. C) Triglycerides are transported to the liver via the hepatic portal vein to be processed. D) Free fatty acids and monoglycerides are assembled into triglycerides before they are absorbed from the lumen of the gut into epithelial cells lining the tract. E) Products of fat digestion are absorbed in the distal portion of the ileum, bound to intrinsic factor.

B

Which of the following statements is TRUE? A) The mother and the fetus exchange nutrients by exchanging blood in the placenta. B) Human chorionic gonadotropin maintains the corpus luteum. C) Human chorionic gonadotropin levels are the highest at the end of pregnancy. D) From the time of implantation through delivery, the corpus luteum is the major source of progesterone. E) Maternal plasma progesterone levels decline in the weeks leading up to the birth of her baby.

B

Which of the following statements regarding protein digestion and absorption is TRUE? A) Only the exocrine pancreas produces enzymes that can digest proteins. B) After absorption, the products of protein digestion are carried by blood directly to the liver. C) Pepsin digests protein mainly in the small intestine. D) The enzymes that digest protein are secreted in active form. E) Free amino acids are the only product of protein digestion that can be absorbed from the lumen into intestinal epithelial cells.

B

Which of the following statements regarding pulmonary surfactant is TRUE? A) It is secreted by type I alveolar cells. B) It increases the compliance of the lungs. C) It increases airway resistance. D) It is secreted into the intrapleural space. E) It can only interact with other lipid molecules.

B

Which of the following tissues is most dependent upon a constant blood supply of glucose? A) Liver B) Brain C) Adipose D) Skeletal muscle E) Cardiac muscle

B

Which of the following would be most likely to cause metabolic acidosis? A) Severe vomiting B) Severe diarrhea C) Hyperventilation D) Hypoventilation E) Traveling to high altitude

B

Which of the following would cause a decrease in the binding affinity of hemoglobin for oxygen? A) Increased pH of the blood B) Increased temperature of the blood C) Decreased DPG levels in erythrocytes D) The presence of carbon monoxide E) Decreased concentration of H+ in the blood

B

Which of these is demonstrated by the oxygen-hemoglobin dissociation curve? A) The greater the PO2 of the blood, the greater the dissociation of O2 from hemoglobin. B) At normal resting systemic arterial PO2, hemoglobin is almost 100% saturated with oxygen. C) At normal resting systemic venous PO2, only about 75% of the hemoglobin is in the form of deoxyhemoglobin. D) More additional oxygen binds to hemoglobin when going from a PO2 of 60 to 100 mmHg, than is added when going from a PO2 of 40 to 60 mmHg. E) As PO2 increases, the saturation of hemoglobin with oxygen increases linearly.

B

Which of these is found in chylomicrons and plasma membranes, and is a precursor for bile salts and steroid hormones? A) Amino acids B) Cholesterol C) Phospholipids D) Monoglycerides E) Protein

B

Which of these results from the second meiotic division? A) Two diploid daughter cells with 46 chromosomes each B) Two haploid daughter cells with 23 chromosomes each C) Two diploid daughter cells with 23 chromosomes each D) A single daughter cell with 46 chromosomes, and a polar body with no genetic material E) Two haploid daughter cells with 46 haploid chromosomes

B

Which statement best describes the manner in which growth hormone stimulates cell proliferation? A) It directly stimulates cell division in most tissues. B) It stimulates the release of insulin-like growth factor I (IGF-1) from the liver and other target tissues of growth hormone. C) It stimulates metabolism. D) It stimulates the release of eicosanoids from lymphoid tissues. E) It stimulates sleep.

B

Which stimulates the secretion of growth hormone? A) Waking from sleep B) Exercise C) Increased levels of IGF-I in blood D) High plasma glucose E) Increased somatostatin secretion

B

You are a scientist at a drug company where your CEO has asked for your team to create a drug that will inhibit spontaneous emptying of the bladder (incontinence). Which of the following drugs would be effective? A) A drug that enhances the effects of parasympathetic neurons on the detrusor muscle B) A drug that blocks the effects of parasympathetic neurons on the detrusor muscle C) A drug that inhibits the effects of sympathetic neurons on the internal urethral sphincter D) A drug that inhibits the effects of somatic neurons on the bladder E) A drug that inhibits the action of sympathetic neurons on the external urethral sphincter

B

You are an EMT and arrive at the scene of an opioid overdose. The overdose victim is breathing extremely slowly and shallowly. When you arrive at the ER, you report that the medical team should watch out for signs of A) metabolic acidosis. B) respiratory acidosis. C) metabolic alkalosis. D) respiratory alkalosis.

B

You are guaranteed to not gain body fat in a diet that is entirely composed of A) carbohydrates. B) None of these answers are correct. C) fats. D) protein.

B

You embark on a hiking trip on Mt. Kilimanjaro (altitude = 19,000 ft; Atmospheric air pressure = 379 mmHg). As you ascend, what happens to the PO2 in the alveoli? A) No change from sea level, as long as we breathe in the same volume of air. B) Alveolar PO2 decreases. C) Alveolar PO2 increases.

B

You embark on a hiking trip on Mt. Kilimanjaro (altitude = 19,000 ft; Atmospheric air pressure = 379 mmHg).mHg). As you ascend, what are you likely to notice about your breathing? A) You need to work to exhale, using your muscles to decrease your thoracic size beyond what is normal B) You need to work harder to inhale, expanding your thoracic size beyond what is normal C) Both would happen D) Neither would happen

B

You have a patient complaining of fatigue and lethargy. She mentions that she recently went on a low-salt, all carbohydrate diet and consumes mostly rice. You notice a swelling in her neck. You order some blood work. Which of the following do you expect? A) Low plasma concentration of thyroid hormones and thyroid-stimulating hormone (TSH) B) Low plasma concentration of thyroid hormones C) High plasma concentration of thyroid-stimulating hormone (TSH) but low thyrotropin-releasing hormone (TRH) D) High plasma concentration of thyroid hormones due to increased secretion of TSH by the pituitary gland E) Low plasma concentration of thyroid hormones and atrophy of the thyroid gland due to reduced concentrations of thyroid-stimulating hormone (TSH)

B

You're studying long-term stress in foster children. One thing you find is that, among the children experiencing the most significant stress for the longest period of time is decreases in ________. A) circulating cortisol levels B) bone and muscle density C) blood glucose levels D) adrenal gland function

B

You've just stepped on the treadmill and are beginning to increase your speed. As you reach your maximum, which of the following values will NOT increase? A) Minute ventilation B) Arterial PCO2 C) Oxygen delivery to muscles D) Oxygen extraction from the blood by muscles E) Body heat production

B

A drug company creates a drug that acts as an antagonist to the human chorionic gonadotropin (hCG) receptor. The effect of this drug, if given in early pregnancy would be A) secretion of large quantities of FSH and LH. B) secretion of large quantities of GnRH. C) degeneration of the corpus luteum and declining levels of progesterone and estrogen. D) secretion of large quantities of progesterone and estrogen. E) secretion of cortisol.

C

A group of students has declared a hunger strike to protest a perception of low-quality dorm food. Jose is one of the protesting students and he has not eaten for three days. Which of the following is likely TRUE of Jose's physiological state? A) The secretion of insulin, glucagon, ACTH, and cortisol are all increased. B) Brain metabolism has shifted to increase utilization of glucose and to decrease oxidation of ketones. C) Muscle protein is broken down to provide substrates for hepatic and renal gluconeogenesis. D) The secretion of insulin, glucagon, ACTH, and cortisol are all decreased. E) The secretion of insulin and cortisol are increased, while glucagon and ACTH secretion are decreased.

C

A patient goes to her doctor, complaining of fatigue, weight gain, and intolerance to cold. She also has a noticeable goiter in her neck. Which of the following is a most likely diagnosis and cause? A) She has hypothyroidism, possibly due to destruction of thyrotrope cells of her anterior pituitary gland. B) She has hyperthyroidism, possibly due to Graves' disease. C) She has hypothyroidism, possibly due to low iodine in her diet. D) She has hyperthyroidism, possibly due to a hypersecreting tumor of the anterior pituitary gland.

C

Acting alone, epinephrine and thyroid hormone each stimulates release of only a small amount of fatty acids from adipose cells. In the presence of thyroid hormone, epinephrine causes a much more substantial release of fatty acids from the cells. What is the term describing the effect of thyroid hormone on epinephrine's actions? A) Antagonistic B) Agonistic C) Permissive D) Direct E) Paracrine

C

After prolonged exertion in a hot climate, baroreceptors would ________ firing, leading to ________ secretion of ________ and thus ________ renal reabsorption of ________. A) increase; increased; renin; increased; Na+ B) decrease; increased; renin; decreased; Na+ C) decrease; increased; vasopressin; increased; water D) increase; decreased; vasopressin; decreased; water E) decrease; decreased; vasopressin; increased; water

C

Agatha's blastocyst implanted on December 5th. She is trying to figure out which date fertilization most likely occurred on. Which is the most likely date of fertilization? A) December 5th B) December 4th C) December 1st D) November 5th E) November 25th

C

Andrew has his gall bladder removed after developing several large gall stones. After surgery, he cannot release large batches of bile into the duodenum, instead, bile is added slowly in small amounts. Which of the following might be TRUE? A) Carbohydrate digestion may be compromised. B) He may become deficient in water-soluble vitamins such as B12. C) He may become deficient in fat-soluble vitamins such as vitamin E. D) He may start to suffer from protein deficiency. E) None of these would occur.

C

At what time in the lifespan is growth hormone secretion rate highest? A) In utero B) During childhood C) During adolescence D) During adulthood E) During senescence

C

Barry's children seem to be struggling often with constipation. To help prevent it, their pediatrician recommends A) avoiding foods that can lead to accumulation of toxins in feces. B) making the children sit on the toilet until they defecate at least once a day. C) feeding the children foods with a high proportion of cellulose and other indigestible carbohydrates. D) feeding the children foods with more lipid content. E) avoiding milk, the children may be lactose intolerant.

C

Compared to the normal plasma osmolarity, the tubular fluid is ________ as it enters Bowman's space, ________ at the beginning of the loop of Henle, ________ at the tip of the loop, and ________ at the beginning of the distal convoluted tubule. A) isosmotic; hyperosmotic; hyperosmotic; isosmotic B) isosmotic; isosmotic; hypoosmotic; hypoosmotic C) isosmotic; isosmotic; hyperosmotic; hypoosmotic D) isosmotic; isosmotic; hypoosmotic; hyperosmotic E) isosmotic; isosmotic; hyperosmotic; isosmotic

C

Deficiency of iodine in the diet results in ________ caused by ________. A) a goiter; increased production of thyroid-stimulating hormone (TSH) and increased thyroid hormone levels B) atrophy of the thyroid gland; loss of negative feedback by thyroid hormones and increased thyroid-stimulating hormone (TSH) levels C) a goiter; loss of negative feedback by thyroid hormones and increased thyroid-stimulating hormone (TSH) levels D) atrophy of the thyroid gland; decreased sensitivity of receptors for thyroid-stimulating hormone (TSH) E) atrophy of the anterior pituitary gland; loss of negative feedback by thyroid hormones

C

During normal metabolism of the human body, about 60% of the energy released from organic molecules A) is used to perform internal work. B) is stored as ATP. C) appears immediately as heat. D) is stored as triglycerides. E) is used to perform external work.

C

During the absorptive state of metabolism A) liver glycogen is broken down to glucose, which is released into the blood. B) glycogen in muscle cells is broken down to glucose, which is used for energy. C) lipoprotein lipase breaks down triacylglycerols in adipose tissue capillaries. D) fatty acids and glycerol are released from adipose tissue. E) lactate and pyruvate secretion into the bloodstream by the muscles increases.

C

During the cephalic phase of gastric stimulation, which of the following does NOT occur? A) Seeing, smelling, and/or tasting food reflexively increase(s) gastric acid secretion. B) Parasympathetic stimulation of secretory cells in the gastric mucosa occurs. C) Gastrin is secreted into the gastric lumen. D) HCl is secreted into the gastric lumen. E) Pepsinogen is secreted into the gastric lumen.

C

During the contractions of gastric emptying A) the stomach is induced to contract by increased sympathetic activity. B) the pyloric sphincter opens to allow stomach contents to pass quickly and completely into the duodenum. C) the pyloric sphincter closes to allow only a small amount of liquefied chyme to pass into the duodenum with each contraction of the stomach. D) the strength of stomach wall contractions is increased when the duodenum secretes cholecystokinin. E) the strongest contractions are in the antrum of the stomach because the intrinsic pacemaker frequency is fastest there.

C

During the early part of the menstrual cycle, ________ acts on granulosa cells, which ________. ________ acts on theca cells, which ________. A) LH; convert androgens to estrogens; FSH; synthesize androgens B) LH; synthesize androgens; FSH; convert androgens to estrogens C) FSH; convert androgens to estrogens; LH; synthesize androgens D) FSH; synthesize androgens; LH; convert androgens to estrogens E) progesterone; synthesize androgens; LH; secrete estrogens

C

Elizabeth has been diagnosed with a gastric ulcer. Carl has been diagnosed with a duodenal ulcer. Both of them are A) treated by interventions that inhibit digestive enzymes. B) caused by hypersecretion of gastric acid. C) treated by interventions that inhibit acid secretion. D) caused by removal of the gallbladder. E) found in the large intestine.

C

Elizabeth suffers from a gastric ulcer. She experiences pain during the times when acid is released into the stomach. Which of the following times is Elizabeth likely to experience the most pain? A) Elizabeth is likely to be in the same amount of pain all the time. B) During the digestion of a high-fat meal, such as greasy French fries. C) During the digestion of a high-protein meal, such as a steak. D) During periods of hunger. E) During digestion of a high-carbohydrate meal, such as a bagel.

C

Following diagnosis with pancreatic cancer, Neil had a large portion of his pancreas removed. All of the following will now be found in lower amounts in Neil's GI tract, except one. Which of the following will NOT be affected? A) Bicarbonate ions B) Amylase C) Bile salts D) Trypsinogen E) Lipase

C

How does the renal countercurrent multiplier mechanism allow the creation of a concentrated urine? A) It transports NaCl from the medullary interstitial fluid into the collecting duct, which directly increases the osmolarity of the urine. B) It transports urea from the medullary interstitial fluid into the collecting duct, which directly increases the osmolarity of the urine. C) By concentrating NaCl in the renal medullary interstitial fluid, it allows water to be reabsorbed from the collecting ducts when vasopressin is present. D) By pumping NaCl and urea into the ascending limb of the loop of Henle, it raises the solute load, which turns into a concentrated urine once water is extracted from the collecting duct. E) When anti-diuretic hormone is present, it stimulates the pumping of NaCl from the medullary interstitial fluid and water follows, concentrating the urine.

C

In the differentiation of a normal female: A) Müllerian inhibiting substance causes the Müllerian ducts to regress. B) The uterus and fallopian tubes are formed from the Wolffian ducts. C) Female genitalia form in the absence of hormonal stimulation. D) The gonads remain undifferentiated throughout fetal life. E) The SRX protein present on X chromosomes makes the gonads differentiate into ovaries.

C

Jerome has had too many alcoholic drinks at a party. One of the effects of alcohol is that it inhibits the release of vasopressin. Which of the following is Jerome likely to notice? A) A reduction in urine volume B) The excretion of glucose in the urine increased C) High volume of dilute urine D) Very concentrated urine E) An increase in blood pressure

C

Julia is a new mother who plans to continue lactation for at least a year after her baby is born. At a post-parturition doctor's visit, the doctor says her blood test indicates declining levels of estrogen and progesterone. Will this interfere with her plans to lactate? A) Yes, high levels of estrogen and progesterone are required for milk production. B) Yes, high levels of estrogen and progesterone are required for milk ejection. C) No, because the secretion of oxytocin and prolactin are more important for lactation. D) No, because even in the absence of hormones, suckling at the breast is sufficient stimulus for milk production.

C

Kim had her gallbladder removed last month and has been experiencing some changes to her digestion. One thing that is particularly noticeable is that she has noticed A) she has trouble digesting high-protein foods such as egg whites. B) she has trouble digesting high-carbohydrate foods such as rice and pasta. C) her feces have a different color. D) she feels more energetic and healthy. E) an increase in her appetite.

C

Maria is a patient in the ward who was recently in a motor vehicle accident. Her injuries are causing her a lot of pain, and as a consequence to the morphine she's been given, her respiratory rate is quite low. Which of the following is likely to be TRUE? A) The pH of her blood has increased. B) The level of O2 in her blood has increased. C) The level of H+ ions in her blood has increased. D) The alveolar CO2 has decreased to near atmospheric levels. E) The alveolar O2 level has increased toward atmospheric levels.

C

Natalia is a few weeks pregnant when she begins to experience pain. Her doctor tells her that she is concerned Natalia may be experiencing an ectopic pregnancy and would like to perform an ultrasound to look for the embryo. Where should Natalia's doctor look? A) Near the cervix B) In the endometrium of the lower half of the uterus C) In the fallopian tubes D) In the endometrium of the upper half of the uterus E) In none of these locations

C

Sarah is a pregnant woman entering her third trimester. She is experiencing a number of physiological changes compared to her non-pregnant state. She checks in with her doctor to make sure everything she's experiencing is normal. Which of these is NOT expected in healthy pregnancy? A) Increased appetite B) Increased blood volume C) Hypoventilation D) All of these are expected E) Cardiac output increases

C

Short-loop negative feedback occurs when hormones from the ________ inhibit hormone secretion by the ________. A) hypothalamus; anterior pituitary B) adrenal cortex; hypothalamus C) anterior pituitary; hypothalamus D) adrenal cortex; anterior pituitary E) anterior pituitary; adrenal cortex

C

Spermatogenesis A) begins with the mitotic division of spermatogonia. B) results in four primary spermatocytes for every spermatogonium. C) results in four spermatozoa for every primary spermatocyte. D) is not complete until after an egg is fertilized, which initiates the second meiotic division. E) takes place completely within the lumen of the seminiferous tubule.

C

The androgen-binding protein functions to A) confer responsiveness of certain cells to male sex hormones. B) transport androgens in the plasma. C) bind and maintain high concentrations of testosterone in the seminiferous tubules. D) transport testosterone across cell walls. E) exert negative feedback on the production of testosterone by the Leydig cells.

C

The corpus luteum persists in pregnancy because the implanting blastocyst secretes: A) large quantities of FSH and LH. B) GnRH, which stimulates the pituitary gland to secrete FSH and LH. C) human chorionic gonadotropin (hCG). D) progesterone and estrogen. E) cortisol.

C

The enzyme lactase A) is involved directly in the digestion of proteins. B) is necessary for the breakdown of milk fats. C) is embedded in the luminal plasma membranes of intestinal epithelial cells. D) promotes the formation of and storage of bile by the gallbladder. E) is overproduced and released by secretory cells in the small intestine in the condition known as lactose intolerance.

C

The onset of puberty is triggered by an increase in A) sensitivity of the gonads to pituitary gonadotropins. B) sensitivity of the pituitary to GnRH. C) GnRH secretion. D) sensitivity of the hypothalamus to negative feedback by sex steroids. E) sensitivity of the pituitary to negative feedback by sex steroids.

C

The principal means of increasing heat production in response to a cold environment is A) vasoconstriction of blood vessels in the skin. B) wearing warm clothing. C) shivering thermogenesis and increased voluntary activity. D) vasodilation of blood vessels in skin. E) increasing food intake to elevate the metabolic rate.

C

The process of spermatogenesis from primary spermatocyte to sperm A) takes about three weeks in the human. B) occurs when a male is an embryo, ceasing at birth. C) requires participation of Sertoli cells. D) occurs in the epididymis. E) occurs as the cells gradually travel between two closely attached Leydig cells.

C

The subjective feeling of hunger is stimulated when A) plasma insulin concentration increases. B) plasma ghrelin concentration decreases. C) plasma leptin concentration decreases. D) plasma glucose concentration increases. E) body temperature increases.

C

The thermoneutral zone is defined as A) the region in the central core of the body where temperature never changes. B) the core body temperature at which no energy is expended to maintain homeostasis. C) the environmental temperature range over which the changes in skin blood flow alone can regulate body temperature. D) the region of the body halfway between the high-temperature core and the low-temperature skin. E) the environmental temperature range in which shivering, changes in blood flow, and changes in thyroid hormone concentrations can maintain homeostasis of the body's core temperature.

C

Tom is a 37-year-old male experiencing fertility issues. In his appointment at the assisted-reproduction clinic he mentions an odd result from a recent genetic test in which he learned he has a mutation on one copy of his gene for androgen-binding protein. Which of the following is likely TRUE of Tom compared to an individual with two functional copies of the gene? A) His cells are less responsive to male sex hormones. B) He is unable to transport androgens in the plasma. C) He has lower concentrations of testosterone in the seminiferous tubules. D) He is unable to transport testosterone across cell walls. E) None of these.

C

Umbilical arteries A) deliver oxygen and nutrient-rich blood from the mother to the placenta. B) deliver oxygen and nutrient-rich blood from the fetus to the placenta. C) carry blood with low oxygen and high metabolic wastes away from the fetus to the placenta. D) carry highly oxygenated blood from the placenta to the fetus. E) carry blood with low oxygen and high metabolic wastes from the placenta to the mother's circulation.

C

What brain center has neurons that fire mainly during inspiration and have input to the spinal motor neurons that activate the diaphragm and inspiratory intercostal muscles? A) The ventral respiratory group of the medulla oblongata B) The pre-Bötzinger complex C) The dorsal respiratory group of the medulla oblongata D) The pneumotaxic center of the pons E) The apneustic center of the pons

C

What enzyme converts chylomicron triacylglycerols into fatty acids and monoglycerides? A) Pancreatic lipase B) Pancreatic amylase C) Lipoprotein lipase D) Pepsin E) Hexokinase

C

What is the definition of a prohormone? A) Hormones that bind to endocrine glands and stimulate the secretion of a second hormone B) Hormones that bind to endocrine glands and inhibit the secretion of a second hormone C) A longer protein or peptide that is cleaved into shorter ones, at least one of which is a peptide or protein hormone D) Hormones that stimulate the expression of receptors for a second hormone, promoting their action E) Steroid hormones that are inactivated by having hydroxyl groups removed from their structure

C

What is the rate-limiting (regulated) step for stimulating the secretion of aldosterone? A) Conversion of angiotensin I to angiotensin II in the blood B) Secretion of angiotensinogen by the liver C) Conversion of angiotensinogen to angiotensin I in the blood D) Secretion of ACTH by the anterior pituitary E) Secretion of angiotensin II by the kidney

C

What region of the nephron reabsorbs about two-thirds of filtered Na+ and Cl-? A) Ascending loop of Henle B) Glomerulus C) Proximal convoluted tubule D) Distal convoluted tubule E) Collecting duct

C

What term describes hormones secreted by the hypothalamus that regulate the secretion of hormones from the anterior pituitary gland? A) Growth factors B) Paracrine factors C) Hypophysiotropic hormones D) Metabotrobic-releasing factors E) Cerebrovitalistic hormones

C

What term describes hormones that influence the secretion of other hormones? A) Mineralocorticoids B) Antagonistic C) Tropic D) Allosteric E) Teratogen

C

When an embryo implants in the wall of the fallopian tube, it is called A) stillbirth. B) erythroblastosis fetalis. C) ectopic pregnancy. D) endometriosis. E) placenta previa.

C

Which correctly describes the cause of the increase in alveolar ventilation that occurs when a person ascends to high altitude? A) The decrease in PCO2 of inspired air decreases alveolar PCO2, stimulating the peripheral chemoreceptors. B) The decrease in PCO2 of inspired air decreases alveolar PCO2, stimulating the central chemoreceptors. C) The decrease in PO2 of inspired air decreases alveolar and arterial PO2, stimulating the peripheral chemoreceptors. D) The decrease in total atmospheric pressure causes a greater negative pressure in the intrapleural space, resulting in deeper and more frequent breathing. E) The decrease in the total barometric pressure at high altitude causes hypocapnea, which stimulates peripheral chemoreceptors.

C

Which does NOT apply to hormones? A) They are chemical regulators that are conveyed from one organ to another via the bloodstream. B) In some cases, the same chemical substances can also function as local regulators and/or neurotransmitters. C) All hormones are derived from cholesterol. D) They are secreted into the blood by ductless glands. E) They are sometimes secreted by neural tissue.

C

Which event occurs during exercise but NOT during fasting? A) Increased breakdown of triglycerides B) Increased glycogenolysis C) Increased glucose uptake by muscle D) Increased fatty acid oxidation E) Increased cortisol secretion

C

Which is TRUE about sperm delivery? A) Most of the ejaculated sperm arrive in the vicinity of the ovum (if there is one). B) Sperm transport through the cervix is made possible by actions of progesterone on the cervical mucus. C) Of the several hundred million sperm in an ejaculation, only about 100 to 200 reach the fallopian tube. D) Sperm are able to fertilize an egg from the instant they arrive in the female reproductive tract. E) Sperm can survive for up to 2 weeks in the mucus of the cervix.

C

Which is TRUE regarding chylomicrons? A) They readily cross the plasma membrane of adipocytes. B) They are converted to monoglycerides and fatty acids by an enzyme inside muscle cells. C) They are converted to monoglycerides and fatty acids by an enzyme found inside capillaries. D) They are formed in the lumen of the GI tract by the action of bile salts. E) They are too large to enter lacteals.

C

Which is a TRUE statement about events occurring in the menstrual cycle? A) Each peak in estrogen concentration is accompanied by an even greater peak in progesterone concentration. B) FSH reaches its highest concentration 3 days before menstruation begins. C) Progesterone is low during the first half of the cycle, and rises to a peak during the second half. D) Ovulation occurs exactly in the middle of the follicular phase of the cycle. E) The corpus luteum normally degenerates on day 14 of the cycle.

C

Which is a result of an absence of thyroid hormones during fetal development? A) Acromegaly B) Cushing's syndrome C) Congenital hypothyroidism (cretinism) D) Graves' disease E) Addison's disease

C

Which is an accurate description of the posterior pituitary and its functions? A) It is glandular tissue and secretes vasopressin and prolactin. B) It is neural tissue and is stimulated to secrete oxytocin and vasopressin by hypophysiotropic hormones. C) It is neural tissue, and vesicles containing oxytocin and vasopressin are transported to axon terminals there. D) It is glandular tissue and releases oxytocin and somatostatin when action potentials arrive along axons from the hypothalamus. E) It is neural tissue that secretes hypophysiotropic hormones that control the secretion of the anterior pituitary hormones.

C

Which is an example of long-loop negative feedback? A) Stimulation of dopamine release by prolactin B) Inhibition of growth hormone-releasing hormone (GHRH) release by growth hormone (GH) C) Inhibition of growth hormone-releasing hormone (GHRH) release by insulin-like growth factor-1 D) Inhibition of corticotropin-releasing hormone by adrenocorticotropic hormone (ACTH) E) Stimulation of thyroid-stimulating hormone (TSH) release by thyrotropin-releasing hormone (TRH)

C

Which is caused by increased levels of the hormone cholecystokinin? A) Contraction of the sphincter of Oddi B) Inhibition of pancreatic enzyme secretion C) Contraction of the gallbladder D) Secretion of HCO3- from the pancreatic duct cells E) Secretion of gastric H+

C

Which of the following describes tubular reabsorption in the kidney? A) The movement of substances from the peritubular capillaries into the tubular fluid B) The movement of substances from the proximal tubule into the loop of Henle C) Transepithelial transport from the lumen of the tubule into renal interstitial fluid D) Movement of Na+, Cl-, and water from glomerular capillaries into Bowman's capsule E) Transport of solutes from the renal medullary interstitial fluid into the collecting duct

C

Which of the following is NOT stimulated by cortisol during stress? A) Gluconeogenesis B) Protein catabolism C) Glucose uptake into muscle cells D) Triglyceride catabolism in adipose cells E) Decreased sensitivity to insulin

C

Which of the following is TRUE during the initial stage of an asthma attack? A) ΔP is increased. B) ΔP is decreased. C) R is increased. D) R is decreased. E) None of these, flow rate is unchanged during an asthma attack.

C

Which of the following molecules crosses the luminal membrane of intestinal cells by facilitated diffusion? A) Glucose B) Lactose C) Fructose D) Galactose E) Maltose

C

Which of the following plays a central role in stimulating the secretion of the hormone gastrin? A) The hormone secretin B) The hormone cholecystokinin (CCK) C) Distention of the stomach D) An increase in [H+] in the lumen of the stomach E) Histamine

C

Which of the following statements about estrogen in females is FALSE? A) In low plasma concentration, estrogen inhibits pituitary secretion of LH and FSH. B) In high plasma concentration, estrogen stimulates pituitary sensitivity to GnRH. C) In the presence of high plasma concentration of progesterone, estrogen stimulates increased secretion of GnRH. D) Estrogen secretion is required for the LH surge. E) Estrogen stimulates follicular growth.

C

Which of the following statements about the response of arteriolar smooth muscle to changing oxygen partial pressures is TRUE? A) Both systemic and pulmonary arterioles respond to a decrease in PO2 by constricting. B) Both systemic and pulmonary arterioles respond to a decrease in PO2 by dilating. C) Systemic arterioles respond to a decrease in PO2 by dilating, but pulmonary arterioles constrict in response to decreased PO2. D) Systemic arterioles respond to a decrease in PO2 by constricting, but pulmonary arterioles dilate in response to decreased PO2. E) Changes in PO2 do not affect arteriolar smooth muscle in the pulmonary system.

C

Which of the following statements concerning secretin is correct? A) The most potent stimulus for secretin secretion is the presence of fat in the duodenum. B) Secretin is the most potent stimulus for pancreatic digestive enzyme secretion. C) Secretin is the most potent stimulus for pancreatic HCO3- secretion. D) Secretin stimulates secretion by the parietal and chief cells. E) Secretin stimulates gastrin secretion by parietal cells.

C

Which of the following statements regarding control of respiration is correct? A) A slight decrease in arterial PO2 is a stronger stimulus for increased ventilation than is a comparable decrease in arterial PCO2. B) The most important signal for regulating ventilation is the H+ concentration of arterial blood. C) Increased concentrations of lactic acid stimulate ventilation primarily by acting on peripheral chemoreceptors. D) An increase in the HCO3- concentration in blood stimulates ventilation. E) At high altitude, a decrease in PCO2 of the blood stimulates an increase in ventilation.

C

Which of the following statements regarding digestion and absorption of carbohydrates is TRUE? A) Carbohydrate digestion begins in the stomach. B) Lactose intolerance results from an insufficiency of the enzyme amylase. C) Digestible polysaccharides are broken down into the monosaccharides glucose, galactose, and fructose, which can be absorbed. D) Sucrose is the main form of carbohydrate that can be absorbed by active transport across the intestinal epithelium. E) Cellulose from plants is a polymer of glucose that can be easily digested and absorbed by the human GI tract.

C

Which of the following statements regarding the control of parturition is FALSE? A) Oxytocin secretion is stimulated by cervical dilation. B) Coordinated uterine contractions cause the cervix to dilate. C) Oxytocin decreases the expression of myometrial estrogen receptors. D) Prostaglandins stimulate myometrial contractions. E) Stretching of the myometrium is one signal that begins uterine contractions.

C

Which of the following substances undergoes renal tubular secretion? A) Ca2+ B) Na+ C) K+ D) H2O

C

Which of the following would increase gluconeogenesis? A) Increased plasma insulin levels B) Increased plasma incretin levels C) Increased plasma cortisol levels D) Decreased plasma growth hormone levels E) Decreased plasma glucagon levels

C

Which of the following would occur if a person lost the ability to synthesize vasopressin? A) The ability to reabsorb water in the proximal tubule would be lost. B) The excretion of glucose in the urine would increase. C) The urine would become hypoosmotic compared to plasma. D) The urine production would decrease dramatically, and the urine osmolarity would be hypertonic compared to plasma. E) Blood pressure would increase significantly.

C

Which of the these events follows the absorption of glucose? A) Glucose is stored as fat in skeletal muscle. B) Glucose is stored as glycogen in adipose tissue. C) Glucose is converted to fat in the liver. D) Glucose is used to make energy by only the brain. E) Glucose is converted to amino acids in the muscles.

C

Which of these causes inhalation/inspiration? A) Increase in the curvature (upward movement) of the diaphragm B) Movement of the ribs closer together due to contraction of the internal intercostal muscles C) Flattening (downward movement) of the diaphragm D) Contraction of the abdominal muscles E) Alveolar pressure increasing above atmospheric pressure

C

Which of these correctly describes the chloride shift? A) In the lungs, chloride enters red blood cells in exchange for bicarbonate ions. B) In the tissues, chloride exits red blood cells in exchange for carbonic acid. C) In the tissues, chloride enters red blood cells in exchange for bicarbonate ions. D) In the lungs, chloride enters red blood cells in exchange for CO2. E) In the tissues, chloride enters red blood cells in exchange for CO2.

C

Which would result from a vitamin D deficiency? A) Hypercalcemia B) An increase in bone mass C) An increase in parathyroid hormone levels D) Increased intestinal absorption of calcium

C

You (a healthy individual) are swimming underwater when you can no longer resist the urge to breathe. This overwhelming urge is probably due to A) the autorhymthic cells in your diaphragm contracting. B) the decrease in O2 available to the cells of the body. C) the increase in plasma H+. D) the increase in pH has made your blood dangerously alkaline.

C

You are working for an educational video company and you've been asked to go to the hospital imaging center to obtain video of segmentation. The imaging center technician asks you where in the GI tract she should image and when during digestion? You tell her that to obtain the best video of segmentation she should record A) the esophagus during swallowing. B) the stomach during digestion of a meal. C) the small intestine as chyme moves past the pyloric sphincter. D) the large intestine during fecal compaction.

C

You have a patient with extensive birth defects that affect the kidney anatomy. Nevertheless, she does have some kidney function. One thing you've noticed in her labs is extremely low renin secretion. Which of the following anatomical locations is likely to exhibit abnormal development? A) The ascending limb of the loop of Henle B) The glomerular capillaries C) The juxtaglomerular apparatus D) The proximal tubule E) The efferent arteriole

C

You have discovered a novel steroid hormone. You'd like to isolate its receptor as part of your next project. Where should you look first? A) On the cell surface B) In the blood stream C) In the cytoplasm D) Any of these might be places where the receptor could be found.

C

You're going hiking and want to pack emergency food in case you get lost or the hike takes longer than you anticipate. Your goal is to pack a food that will have the biggest increase on your blood glucose level to be able to supply your hiking muscles with glucose for cellular respiration. Which of the following foods will lead to the biggest and most immediate increase in blood glucose? A) A snack rich in lipids, such as slices of cheese B) A snack rich in protein, such as a dried meat stick C) A snack rich in digestible polysaccharides such as a cooked potato D) All of these will have equal impacts on blood glucose level E) A snack rich in cellulose, such as a bag of celery

C

You're resting on the couch watching TV. Which of the following is the most likely reflection of the values in your body? Regarding the partial pressures of O2 and CO2, which of the following statements is true in a normal person at rest? A) Atmospheric PO2 is approximately 100 mmHg and the alveolar PO2 is approximately 160 mmHg. B) The PCO2 in the air is approximately 40 mmHg and the alveolar PCO2 is approximately 60 mmHg. C) The PO2 in the systemic arteries is approximately 100 mmHg and the alveolar PO2 is approximately 105 mmHg. D) PCO2 in the systemic veins is less than systemic arterial PCO2. E) PCO2 in the pulmonary arteries is less than pulmonary venous PCO2.

C

A bacterium that has been associated with ulcers is A) Eschericia coli. B) Clostridium difficile. C) Staphylococcus aureus. D) Helicobacter pylori. E) Streptococcus pyogenes.

D

A chemical precursor for cortisol is ________, which is made from ________. Cortisol is secreted by ________ when stimulated by ________ from the anterior pituitary. A) androstenedione; progesterone; adrenal cortex; corticotropin-releasing hormone (CRH) B) progesterone; cholesterol; adrenal medulla; adrenocorticotropic hormone (ACTH) C) testosterone; cholesterol; adrenal medulla; adrenocorticotropic hormone (ACTH) D) progesterone; cholesterol; adrenal cortex; adrenocorticotropic hormone (ACTH) E) estradiol; cholesterol; adrenal medulla; corticotropin-releasing hormone (CRH)

D

A man with hypertension takes a diuretic that is not potassium-sparing, and he does not increase his ingestion of potassium. Which of the following side-effects would the drug be most likely to cause? A) Increased blood volume B) Depolarized neuronal cell membranes C) Hyperkalemia D) Cardiac arrhythmia E) All of these side effects would occur.

D

Action potentials in neurons in the inspiratory center of the medulla oblongata result in action potentials in the ________ nerves to the diaphragm, which in turn cause ________ of the muscle, resulting in a/an ________ in the volume of the thoracic cage. A) motor; contraction; decrease B) parasympathetic; contraction; decrease C) sympathetic; contraction; increase D) motor; contraction; increase E) sympathetic; relaxation; increase

D

After CO2 is produced by tissues, in what form is most of that carbon transported to the lungs for removal from the body? A) As dissolved CO2 B) Bound to hemoglobin C) As H2CO3 D) As dissolved HCO3- E) As carbonic anhydrase

D

Baby Sylvia is born with a thyroid deficiency. Which of the following can we expect for Sylvia if it goes untreated? A) Gigantism B) Acromegaly C) Growth rate will be faster than normal. D) Growth rate will be slower than normal. E) Body temperature will be above normal.

D

Beginning on the first day of the menstrual cycle, the order of phases in the uterus is A) proliferative, secretory, menstrual. B) secretory, proliferative, menstrual. C) menstrual, secretory, proliferative. D) menstrual, proliferative, secretory. E) secretory, menstrual, proliferative.

D

Bicarbonate is A) secreted into the lumen by gastric epithelial cells and into the interstitial fluid by pancreatic duct cells. B) secreted into the lumen by pancreatic acinar (exocrine) cells, and into the interstitial fluid by pancreatic duct cells. C) secreted into the lumen by pancreatic duct cells, and into the interstitial fluid by pancreatic acinar (exocrine) cells. D) secreted into the interstitial fluid by gastric epithelial cells, and into the lumen by pancreatic duct cells. E) secreted into the lumen by both gastric epithelial cells and pancreatic duct cells.

D

Ejaculation of semen from the penis is caused by A) a parasympathetic reflex that contracts the seminal vesicles, epididymis, and vas deferens. B) a somatic reflex that initiates sperm production and motility, and contracts skeletal muscle surrounding the vas deferens. C) voluntary relaxation of the internal urethral sphincter, and sympathetic stimulation of smooth muscle contraction in the walls of the ureters and the urethra. D) sympathetic nerve firing, which initiates smooth muscle contraction in the epididymis, vas deferens, ejaculatory ducts, prostate, and seminal vesicles. E) parasympathetic stimulation of smooth muscle in the ureters, urinary bladder, and urethra.

D

Endogenous pyrogen A) is released by infection-fighting leukocytes and acts on muscle tissue. B) is released by the hypothalamus and acts on infection-fighting leukocytes. C) is released by infection-fighting leukocytes and has the same action as aspirin. D) is released by infection-fighting leukocytes and acts on the temperature-integrating centers of the hypothalamus. E) is released by the pyloric region of the stomach and raises the temperature of digestion.

D

Following a series of high-intensity training runs, lactic acid was generated by the leg muscles and is now circulating in the blood. Which of the following is likely to occur? A) Decreased renal secretion of hydrogen ion B) Decreased renal reabsorption of bicarbonate ion C) Increased plasma PCO2 D) Decreased urinary pH E) Decreased urinary ammonium

D

Ghrelin is secreted by ________, and increases ________ while decreasing ________. A) cells in the stomach; neuropeptide Y secretion; hunger B) adipose tissue; growth hormone secretion; leptin secretion C) muscle tissue; hunger; growth hormone secretion D) cells in the stomach; hunger; fat breakdown E) adipose tissue; hunger; fat breakdown

D

Glucagon secretion is stimulated by A) hypoglycemia and parasympathetic nerve firing. B) hyperglycemia and a high concentration of epinephrine. C) high plasma epinephrine and high plasma incretin concentrations. D) hypoglycemia and high plasma epinephrine concentration. E) hyperglycemia and high plasma incretin concentration.

D

How does estradiol increase the likelihood that an egg will be fertilized? A) It inhibits the motility of uterine smooth muscle. B) It stimulates inhibin levels, which causes a surge of FSH. C) It increases libido, or sex drive, in women. D) It changes cervical mucus to a watery consistency that allows sperm to penetrate the cervix. E) It stimulates LH receptors that help maintain a corpus luteum.

D

How is the concentration of a hormone in plasma determined? A) Only by its rate of secretion B) By the number of its target cells in the body C) Only by its rate of synthesis D) By its secretion and clearance rates, and whether or not it binds to carriers and/or other plasma proteins E) Only by the rate of its degradation by the liver and kidneys

D

If all other factors remain the same, which of these results in an increase in the amount of oxygen bound to hemoglobin? A) Increased plasma [H+] B) Increased plasma temperature C) Increased plasma [2,3 diphosphoglycerate] D) Increased plasma PO2 E) Increased plasma [carbon monoxide]

D

In the embryonic differentiation of a normal male A) undifferentiated gonads develop into testes during the fifth week of embryonic life. B) the Wolffian ducts regress. C) Mullerian inhibiting substance is absent. D) testosterone directly stimulates development of the epididymis, vas deferens, and seminal vesicles. E) testosterone directly stimulates development of the penis and scrotum.

D

In what form is the body's largest pool of calcium? A) Embedded in collagen in many types of connective tissue B) Blood, dissolved within the plasma C) Skeletal muscle, stored in terminal cisternae D) Bones, in the form of hydroxyapatites E) Liver, inside the endoplasmic reticulum

D

Regarding the relationships between the rates of oxygen consumption/carbon dioxide production and alveolar ventilation, which of these statements is TRUE? A) Increased ventilation without a similar increase in metabolism is called hypoventilation. B) In hypoventilation, alveolar PCO2 decreases below resting levels. C) In hyperventilation, alveolar PCO2 can decrease to zero. D) A decrease in metabolism without a similar decrease in alveolar ventilation would result in hyperventilation. E) An increase in ventilation with a matching increase in metabolism is called hyperventilation.

D

Sarah has been diagnosed with pancreatic cancer. The first treatment is to remove 90% of her pancreatic tissue surgically. Due to the loss of exocrine tissue, which of the following foods will Sarah have more difficulty digesting post-surgery? A) Lipids B) Proteins and peptides C) Starches D) All of these

D

Sarah is a type 1 diabetic who forgot to bring her insulin today. By the end of the day, Sarah is not feeling well. She stops at the campus health center on her way back to her dorm, where they run a few tests on her blood. Which of these test results is likely? A) Low plasma ketone levels B) Increased blood volume C) Decreased oxygen levels in Sarah's blood D) Hyperglycemia E) Decreased blood pressure

D

Sarah is taking antacids every day. This habit is raising her stomach and duodenal pH close to neutral. Which of the following enzymes may have decreased activity? A) Amylase B) Lipase C) Trypsin D) Pepsin E) Chymotrypsin

D

Sexual differentiation of the embryo is normally determined by A) the number of chromosomes, with males having 22 pairs and females having an extra pair of XX chromosomes. B) the presence of two X chromosomes, which causes the primordial testes to degrade. C) the presence of one Y chromosome, which causes the primordial ovaries to degrade. D) the presence of the Y chromosome, which directs the undifferentiated gonads to develop into testes (when there is one X chromosome present as well). E) hormones produced by the placenta.

D

The acrosome reaction is triggered by A) deposition of sperm into the female reproductive tract. B) capacitation of sperm. C) estrogen. D) binding of the heads of sperm to the zona pellucida. E) the first meiotic reduction division of the ovum.

D

The amount of a substance that is excreted in the urine is equal to the amount that is ________ plus the amount that is ________ minus the amount that is ________. A) filtered; reabsorbed; secreted B) reabsorbed; filtered; secreted C) secreted; reabsorbed; filtered D) filtered; secreted; reabsorbed E) reabsorbed; secreted; filtered

D

The main source of enzymes that cleave disaccharides into monosaccharides is A) the pancreas. B) the liver. C) gastric secretory cells. D) the luminal membrane of epithelial cells in the small intestine. E) the salivary glands.

D

The major source of estradiol and progesterone in a non-pregnant woman is the A) thyroid gland. B) adrenal gland. C) testis. D) ovary. E) uterus.

D

The placenta is A) a site where maternal blood enters the fetal circulation. B) a site formed by the proliferation of the maternal endometrium and myometrium. C) a site where fetal blood can enter the maternal circulation. D) a site of diffusion and transport of wastes, oxygen, and nutrients between fetal and maternal blood. E) formed from maternal circulatory stem cells, and serves as an exchange organ between mother and fetus.

D

The primary integrator of body temperature control reflexes is the A) medullary cardiovascular center. B) cerebral cortex. C) thyroid gland. D) hypothalamus. E) adrenal medulla.

D

Under what condition is lung compliance increased? A) Respiratory distress syndrome of the newborn B) Prolonged shallow breathing C) Cystic fibrosis D) Emphysema E) Asthma

D

What is a main difference between the modes of action of peptide hormones and steroid hormones? A) Peptide hormones bind to intracellular receptors, whereas steroid hormones bind to receptors on the cell surface. B) Peptide hormones act as second messengers, whereas steroid hormones bind to receptors in the cytosol. C) Peptide hormones bind to receptors on the cell surface, whereas steroid hormones act as second messengers. D) Peptide hormones bind to receptors on the cell surface, whereas steroid hormones bind to intracellular receptors. E) There are no differences; both act by binding to intracellular receptors.

D

What would be the effect of cutting the fallopian tubes at their midpoints? A) Sperm would not travel from the vagina to the uterus. B) Sperm would not travel from the testes to the epididymides. C) Estrogen would not enter the bloodstream. D) Ova would not travel from the ovaries to the uterus. E) The Wolffian duct system would be disconnected from the rest of the reproduction system.

D

When you head to bed at night, you bring a glass of tap water with you and set it on the nightstand. Upon waking in the morning, you discover there are tiny gas bubbles along the sides of the container. The best explanation for this is A) the concentration of a particular gas in a liquid equilibrates to match the concentration of that gas in the air to which the liquid is exposed. B) different gasses have the same solubility in liquids. C) the partial pressure of a gas in a liquid is equal to the amount freely diffusing in the liquid plus the amount bound to large molecules within the liquid. D) the partial pressure of a particular gas in a liquid equilibrates to match the partial pressure of that gas in the air to which the liquid is exposed. E) as the partial pressure of a particular gas in an air sample increases, the concentration of that gas in a liquid exposed to that air sample decreases.

D

Which is NOT a function of Sertoli cells? A) Forming a barrier between the blood and developing germ cells B) Secreting inhibin C) Nourishing developing sperm D) Converting estrogens into testosterone E) Phagocytizing defective sperm

D

Which is NOT a transport mechanism typically seen in renal tubular epithelial cells? A) Cotransport proteins in the luminal membrane of the proximal tubule that move Na+ and glucose from the proximal tubule into epithelial cells B) Transport proteins that move glucose by facilitated diffusion from inside of proximal tubule cells into the renal interstitial fluid C) Ion channels that allow Na+ to move by diffusion from the lumen of the proximal tubule into epithelial cells D) Na+-K+ ATPase pumps in the luminal membrane of proximal tubule epithelial cells that move Na+ from inside the cell into the tubule and K+ from the tubule lumen into the cell E) Countertransport proteins that move Na+ into proximal tubule epithelial cells while moving H+ from the cells into the lumen

D

Which is TRUE about hormonal control during pregnancy? A) The corpus luteum must remain functional for 38 weeks because it is the main source of estrogen and progesterone. B) The placenta responds to estrogen and progesterone from the corpus luteum, but does not secrete them. C) The placenta produces estrogen without any contribution from the fetal adrenal cortex. D) The placenta is provided with androgens by the maternal ovaries and adrenal glands, and by the fetal adrenal glands. E) The placenta secretes only steroid hormones, while fetal and maternal pituitary glands secrete pregnancy-specific protein hormones.

D

Which is TRUE about the maturation of ovarian follicles? A) Granulosa cells produce androgens, which stimulates follicle maturation. B) Theca cells produce estrogen, which is converted to the androgens that stimulate follicle maturation. C) On day 1 of the cycle, growing follicles have both FSH and LH receptors on granulosa cells. D) Growing follicles produce estrogen, which further stimulates follicle maturation. E) Follicle maturation is stimulated when FSH binds to theca cells and LH binds to granulosa cells.

D

Which is a function of the gonadotropic hormones? A) Stimulating the release of follicle-stimulating hormone (FSH) and luteinizing hormone (LH) B) Inhibiting the release of growth hormone C) Inhibiting the release of prolactin D) Inducing the secretion of steroid hormones by the gonads in both males and females E) Stimulating the release of gonadotropin-releasing hormone (GnRH)

D

Which occurs mainly in the small intestine? A) Gastrin is secreted. B) H+ is secreted from epithelial cells into the lumen. C) Pepsinogen is activated. D) Acidic chyme is neutralized. E) Bile salts are manufactured.

D

Which of the following hormones is NOT secreted by the anterior pituitary gland? A) Somatotropin B) Prolactin C) Corticotropin D) Erythropoietin E) Thyroid-stimulating hormone (TSH)

D

Which of the following is NOT a fate of absorbed glucose? A) It is converted to glycogen by liver cells. B) It is converted to fatty acids and alpha-glycerol phosphate in adipose cells. C) It is converted to glycogen in muscle cells. D) It is converted to urea by liver cells. E) It is converted to fatty acids and alpha-glycerol phosphate by liver cells.

D

Which of the following is most responsible for keeping the lung surface and the thoracic wall from separating? A) The presence of surfactant B) The negative pressure in the alveolar space C) The force exerted by the external intercostal muscles D) Subatmospheric intrapleural pressure E) Alveolar pressure is higher than atmospheric pressure

D

Which of the following pathways is activated during the cephalic phase of gastrointestinal control? A) Sympathetic nerves to enteric nervous system B) Secretion of cholecystokinin C) Secretion of secretin D) Parasympathetic nerves to enteric nervous system E) Short reflexes between the small intestine and stomach

D

Which of the following statements about hormone synthesis is NOT correct? A) Progesterone can be a precursor of cortisol. B) Progesterone is a precursor of mineralocorticoid and glucocorticoid hormones. C) Pregnenolone is a precursor for all steroid hormones. D) Tryptophan is a precursor for thyroid hormones. E) Peptide hormones are polymers of amino acids.

D

Which of the following statements about pancreatic enzymes is FALSE? A) Trypsinogen is secreted in an inactive form. B) Pancreatic amylase digests carbohydrates. C) Pancreatic lipase digests fats. D) Except for trypsinogen, other proteolytic enzymes are secreted in active form. E) Trypsinogen is activated by enterokinase.

D

Which of the following statements regarding endocrine regulation of breast function in females is FALSE? A) During puberty, ductal growth is stimulated by increasing estrogen levels in plasma. B) During puberty, progesterone stimulates the growth of breast alveoli. C) During pregnancy, the main source of plasma prolactin is secretion from the maternal anterior pituitary gland. D) Prolactin is the main hormone mediating the milk ejection reflex. E) Breast development fluctuates with changing blood concentrations of estrogen and progesterone during each menstrual cycle.

D

Which of the following statements regarding male reproductive function is FALSE? A) Sperm formed in the testes enter the epididymis before entering the vas deferens. B) The seminal fluid is alkaline and rich in nutrients. C) The Sertoli cells produce androgen-binding protein. D) Sperm entering the epididymis are fully motile. E) Most of the seminal volume is provided by the secretions of the seminal vesicles and prostate gland.

D

Which of the following statements regarding renal handling of Na+ is correct? A) In the proximal tubule, Na+ is actively transported across the luminal membrane of the tubular epithelial cells. B) Atrial natriuretic factor increases Na+ reabsorption. C) In the absence of aldosterone, Na+ will be secreted by the cortical collecting ducts. D) Na+ is actively reabsorbed in the ascending limb of the loop of Henle. E) Without vasopressin, the collecting duct is impermeable to Na+.

D

Which of these bests describes colostrum? A) It is composed of dark-colored solid waste ejected from a fetus's bowels shortly after it is born. B) It is a hormone secreted by the hypothalamus that inhibits prolactin secretion. C) It is the watery remnant of breast milk that continues to be expelled from the breasts after a baby is weaned. D) It is a watery, protein-rich fluid secreted from maternal breasts after delivery of a baby. E) It is an estrogen-rich fluid that fills the antrum of a mature ovarian follicle.

D

Which of these would tend to increase ventilation? A) Lower than normal arterial PCO2 B) Higher than normal arterial pH C) Breathing carbon monoxide D) Breathing air with increased PCO2 E) Iron-deficiency anemia

D

Which structure is NOT part of the "anatomical dead space"? A) Bronchiole B) Trachea C) Bronchus D) Respiratory bronchiole E) Terminal bronchiole

D

Which would result in the greatest stimulation of aldosterone secretion? A) Increasing plasma K+ concentration and decreasing plasma angiotensin II concentration B) Decreasing plasma K+ concentration and increasing plasma angiotensin II concentration C) Increasing plasma Na+ concentration and increasing plasma volume D) Increasing plasma K+ concentration and increasing plasma angiotensin II concentration E) Decreasing plasma Na+ concentration and decreasing plasma angiotensin II concentration

D

You are a new pediatrician at an office where you've inherited cases from a previous doctor and are sitting down to go through case files. Today you'll be seeing Trevor, a toddler who has been diagnosed with a disease called megacolon. Back in medical school you learned that megacolon can be characterized by partial paralysis of the the nerves found within the wall of the GI system. Which of the following is likely to be among Trevor's symptoms? A) Incomplete denaturation of ingested proteins B) Discolored feces due to lack of Bile C) Diarrhea D) Constipation and a swollen-looking belly E) Abnormal blood glucose regulation

D

You are the CEO of a drug company where you've asked five teams of scientists to generate new drugs to decrease blood pressure. You can only send four of the drugs to market. Which of these drugs is least likely to decrease blood pressure and can be eliminated from potential development? A) A drug that interferes with aldosterone synthesis B) A drug that is an agonist of atrial natriuretic factor C) A drug that decreases sympathetic stimulation of renal arterioles D) A drug that enhances the activity of angiotensin-converting enzyme E) A drug that decreases liver production of angiotensinogen

D

You go see a scary movie. By the end of the movie your heart is racing thanks to high levels of epinephrine in your bloodstream. You ate nothing during the movie, but your blood glucose level is likely A) low due to an overall lack of nutrients. B) low due to the action of cortisol on blood glucose regulation. C) elevated due to healthy post-absorptive state mechanisms. D) elevated due to the action of epinephrine on blood glucose regulation. E) elevated due to the pasta meal consumed at home before the movie.

D

You have discovered a novel hormone. Early experiments have shown that it is quite hydrophobic. Which of the following is likely to also be TRUE of these hormones? A) They mostly bind to receptor proteins in the surface membrane of target cells. B) They are generally polar molecules. C) They usually have very rapid effects on target cells. D) Their mechanism of action generally involves altering protein synthesis. E) They are highly soluble in blood plasma.

D

You play a trick on your little brother and replace all the sugar in the sugar bowl with salt. In the morning, you find him eating a bowl full of cereal with salt on it. You realize he's eaten quite a bit of salt! Which is likely to occur? A) Urine osmolarity would decrease. B) Atrial natriuretic factor secretion would decrease. C) Firing by hypothalamic osmoreceptors would decrease. D) Secretion of atrial natriuretic factor would increase. E) Secretion of vasopressin would decrease.

D

Your patient has advanced stage cancer and her liver is starting to fail. One consequence is markedly reduced plasma proteins. In what way might this affect her endocrine system? A) Transport of all hormones in the blood is compromised B) No change to endocrine system function C) A reduction in the synthesis of steroid hormones by the liver D) A reduction in the rate of hydrophobic hormones like steroid and thyroid hormones reaching their target tissues E) A reduction in the rate of peptide-type hormones reaching their target tissues

D

A deficiency in dietary iodine can cause A) congenital hypothyroidism (cretinism). B) a slowing of mental functions. C) a low metabolic rate. D) a goiter. E) All of the choices are correct.

E

A hormone may be A) inactivated by its target cell. B) activated by its target cell. C) inactivated by nontarget cells. D) excreted before it has a chance to act on a target cell. E) All of the choices could be correct.

E

A patient appears in a clinic complaining of irritability and chronic sweatiness. He also has a lump in his neck near his larynx. Blood tests show he has low levels of thyroid-stimulating hormone (TSH) in his plasma. Which of the following is the most likely diagnosis? A) Primary hypothyroidism B) An iodine-insufficient diet C) Secondary hypothyroidism D) Hyposecretion of TRH E) Graves' disease

E

Acclimatization to a hot environment includes which of these physiological changes? A) A decrease in the set point temperature regulated by the hypothalamus B) Decreased volume and increased salt concentration of sweat glands C) Decreased volume and salt concentration of sweat from the sweat glands D) Increased volume and salt concentration of sweat from the sweat glands E) Increased volume and decreased salt concentration of sweat from the sweat glands

E

After a day of fasting during Ramadan, Amira's blood Na+ level is quite low. Which of the following is likely to occur? A) An increase in plasma volume, which induces an increase in GFR and Na+ reabsorption rate B) A decrease in plasma volume, which induces an increase in GFR and Na+ reabsorption rate C) An increase in plasma volume, which induces an increase in GFR and a decrease in Na+ reabsorption rate D) A decrease in plasma volume, which induces a decrease in GFR and Na+ reabsorption rate E) A decrease in plasma volume, which induces a decrease in GFR and an increase in Na+ reabsorption rate

E

All of the following substances are present in proximal tubular fluid in the kidney, but which one is NOT normally present in urine? A) Ca2+ B) H+ C) K+ D) HPO42- E) Glucose

E

Bile is synthesized and secreted by the A) gallbladder. B) gastric mucosa. C) pancreas. D) duodenum. E) liver.

E

Carl has hypothyroidism and therefore his Na+/K+ ATPase pumps are functioning at a reduced rate. Will there be any effect on the transport of Na+? A) There will be a reduction in Na+ transport in Bowman's capsule epithelial cells. B) There will be a reduction in ATP-dependent, Na+ transport on the apical side of the tubule epithelial cells. C) There will be a reduction in Na+ secretion on both sides of the tubule epithelial cells. D) There will be no change to Na+ transport. E) There will be a reduction in Na+ transport on the basolateral side of the tubule epithelial cells.

E

Following diagnosis with pancreatic cancer, Neil had a large portion of his pancreas removed. Which of the following hormones will now be found in lower amounts? A) Vasopressin B) Thryoid hormone C) CCK D) Oxytocin E) Insulin

E

Following strenuous exercise, blood levels of lactic acid are ________ compared to pre-exercise levels. This change in acid concentration causes a reflex ________ in ventilation by activating ________. A) decreased; decrease; peripheral chemoreceptors B) decreased; decrease; central chemoreceptors C) increased; increase; baroreceptors in the aortic arch D) increased; increase; central chemoreceptors E) increased; increase; peripheral chemoreceptors

E

In untreated type 1 diabetes mellitus (insulin-dependent diabetes mellitus), acidosis occurs because A) dehydration concentrates the blood so even though H+ production is normal, its concentration is high. B) kidney failure results in a decreased ability to excrete the normal amount of H+. C) excess plasma glucose is taken up by the liver and converted to fatty acids. D) insulin levels are extremely high, and insulin is an acidic protein hormone. E) excessive lipolysis results in increased plasma fatty acid levels, which the liver converts to acidic ketones.

E

In which region of the nephron does the fractional reabsorption of water vary the most in response to variation in the state of hydration? A) The glomerulus B) The proximal convoluted tubule C) The loop of Henle D) The distal convoluted tubule E) The collecting duct

E

In which region(s) of the GI tract do peristaltic contractions occur? A) The small intestine B) The large intestine C) The stomach D) The esophagus E) All of these options are correct.

E

Iron is stored in the body mainly as a protein-iron storage complex called A) glycogen. B) hemochromatosis. C) myoglobin. D) transferrin. E) ferritin.

E

Julia has just been diagnosed with anemia. Oddly, during her appointment her doctor kept talking about her digestive system and some item that was not being produced in adequate amounts. Julia can't remember what digestive secretion is to blame for her anemia. Which of the following is the most likely culprit? A) HCl B) Lipase C) Bile D) Amylase E) Intrinsic factor

E

Sarah is a type 1 diabetic who forgot to bring her insulin today. By the end of the day, Sarah is not feeling well. Which would NOT be a characteristic of Sarah at the end of the day with no insulin injections? A) Decreased glycogen stores in muscles B) Increased glucose-bound hemoglobin C) Decreased blood pH D) Increased urination frequency E) Decreased thirst

E

Sarah is a type 1 diabetic. She recently refilled her prescription, but was given glucagon instead of insulin. After her first glucagon injection, which of the following is likely TRUE? A) Increased glucose uptake by muscle cells B) Increased triglyceride synthesis in adipose tissue C) Decreased glycogenolysis in muscle D) Decreased gluconeogenesis in liver E) Increased glycogenolysis in liver

E

Segmentation of the small intestine A) occurs only between meals, and functions to stimulate the secretion of gastrin and the resulting mass movements that empty the large intestine. B) occurs at a rate of 3 contractions per minute throughout the small intestine, although the strength of contractions is much greater in the ileum, which has a much thicker layer of smooth muscle in its walls. C) functions mainly to prevent any forward movement of chyme, so that all digestion and absorption is complete before undigestible materials move into the jejunum. D) are peristaltic movements that sweep undigestible materials out of the small intestine between meals. E) mixes chyme with digestive enzymes, brings food molecules near the wall for absorption, and slowly moves small intestine contents toward the cecum.

E

Susan recently had bariatric surgery which removed 85% of her stomach tissue and left her with a much reduced stomach volume. Which of the following is TRUE of Susan post-surgery? A) She is not able to completely digest proteins and amino acids. B) Her chyme will tend to become more acidic than before the surgery. C) She will have a reduced ability to absorb water from her meals. D) She will have a reduced ability to form her food into a bolus. E) She will be able to store less ingested foods and will require smaller, more frequent meals.

E

The follicular phase of the ovarian cycle coincides with which phase or phases of the uterine cycle? A) Both the menstrual phase and the secretory phase B) The secretory phase C) Both the follicular phase and the secretory phase D) Both the proliferative phase and the secretory phase E) Both the menstrual phase and the proliferative phase

E

The time span over which sexual intercourse is most likely to result in conception is from ________ day(s) before ovulation to ________ day(s) after ovulation. A) 0; 3 B) 1; 1 C) 2; 4 D) 1; 5 E) 5; 1

E

What is the major hormone responsible for mediating the body's general response to stress? A) Thyroid hormone B) Growth hormone C) Testosterone D) Aldosterone E) Cortisol

E

Which accurately describes lipase? A) It is mainly produced in the liver, and secreted into the small intestine. B) It emulsifies lipids. C) It is secreted by the endocrine pancreas. D) It is produced in the exocrine pancreas, and catalyzes the degradation of chylomicrons into proteins and fats. E) It catalyzes the breakdown of triglycerides into monoglycerides and free fatty acids.

E

Which accurately describes the metabolic actions of cortisol during fasting? A) It decreases blood glucose by increasing the sensitivity of muscle and adipose tissue to insulin. B) It stimulates protein synthesis in muscle cells, and is permissive of triglyceride synthesis by adipocytes. C) It increases sensitivity of muscle and adipose to insulin, and inhibits gluconeogenesis in the liver. D) It directly stimulates gluconeogenesis and lipolysis, and increases sensitivity of muscle and adipose tissue to insulin. E) It is permissive of gluconeogenesis and lipolysis, and reduces sensitivity of muscle and adipose tissue to insulin.

E

Which are hormones secreted by the posterior pituitary? A) Vasopressin and dopamine B) Corticotropin and dopamine C) Oxytocin and prolactin D) Vasopressin and corticotropin E) Antidiuretic hormone (ADH) and oxytocin

E

Which correctly describes a basic renal process? A) Fluid is filtered from Bowman's capsule into the glomerulus. B) Substances are secreted from the tubule into the peritubular capillaries. C) Substances are reabsorbed from the peritubular capillaries into the tubular lumen. D) Substances are actively secreted from glomerular capillaries into Bowman's capsule. E) Fluid moves by bulk flow from glomerular capillaries into Bowman's space.

E

Which correctly describes the composition of the glomerular filtrate? A) It is identical to urine, but has a much smaller flow rate. B) It is identical to urine, but has a much larger flow rate. C) It is identical to blood plasma, except it lacks red blood cells. D) It is similar to plasma, except it does not contain glucose. E) It is similar to plasma, except it does not contain plasma proteins.

E

Which equation is equal to the net glomerular filtration pressure? A) GFP = hydrostatic pressure in glomerular capillaries - hydrostatic pressure in Bowman's capsule - osmotic pressure due to protein in Bowman's capsule B) GFP = osmotic pressure due to protein in plasma - hydrostatic pressure in glomerular capillaries - hydrostatic pressure in Bowman's capsule C) GFP = hydrostatic pressure in glomerular capillaries + hydrostatic pressure in Bowman's capsule + osmotic pressure due to protein in plasma D) GFP = hydrostatic pressure in glomerular capillaries + hydrostatic pressure in Bowman's capsule - osmotic pressure due to protein in plasma E) GFP = hydrostatic pressure in glomerular capillaries - hydrostatic pressure in Bowman's capsule - osmotic force due to proteins in plasma

E

Which is NOT a function performed by saliva? A) Moistening and lubricating food for swallowing B) Starch digestion C) Enabling the sense of taste D) Killing bacteria E) Emulsifying lipids

E

Which is NOT an endocrine function performed by the liver? A) Secreting insulin-like growth factor 1 B) Clearing hormones from plasma C) Secreting angiotensinogen D) Producing plasma proteins that bind hormones E) Secreting insulin

E

Which is NOT stimulated by the hormone insulin? A) An increase in the number of glucose transporters in cell membranes B) An increase in amino acid uptake by cells C) An increase in glycogen synthesis D) An increase in triglyceride synthesis E) An increase in ketone synthesis

E

Which is NOT true about receptors for steroid hormones? A) They may be proteins found in the nucleus. B) They undergo allosteric modulation when they bind to the hormone. C) They regulate gene transcription. D) They may be found in the nucleus. E) They are synthesized from cholesterol.

E

Which is NOT true about receptors that mediate digestive reflexes? A) They are located in the gastrointestinal tract wall. B) They include chemoreceptors, osmoreceptors, and mechanoreceptors. C) They may relay information to integrative centers in the CNS or to the enteric plexuses. D) They may be endocrine cells. E) They only activate feedforward pathways.

E

Which is NOT true about the countercurrent multiplier system of the kidney? A) It creates a hyperosmolar medullary interstitial fluid that allows the kidneys to form hypertonic urine. B) The descending loop of Henle is permeable to water. C) There is active transport of sodium and chloride out of the ascending limb of the loop of Henle. D) The ascending loop of Henle is not permeable to water. E) The fraction of filtered NaCl reabsorbed from the ascending limb equals the fraction of filtered water reabsorbed from the descending limb.

E

Which is TRUE during the luteal phase of the menstrual cycle? A) Both estrogen and progesterone concentrations are kept low by negative feedback. B) Estrogen concentration remains low, while progesterone concentration rises to a peak. C) Progesterone concentration remains low, while estrogen concentration rises to a peak. D) Progesterone concentration in the plasma is high, but estrogen concentration rises even higher. E) Estrogen concentration in the plasma is high, but progesterone concentration rises even higher.

E

Which is true about the growth of long bones? A) Osteoblasts dissolve bone tissue when bones remodel in response to sex hormones. B) Osteoclasts are responsible for depositing new bone tissue at the epiphyseal growth plate before puberty. C) Before adolescence, bones are completely made up of cartilage; after puberty they ossify and harden. D) Growth in length occurs at a single epiphyseal growth plate in the center of the shaft of long bones. E) Growth in length occurs at two epiphyseal growth plates near the ends of long bones.

E

Which of the following best describes a genetic male (XY) who lacks functional androgen receptors (androgen insensitivity syndrome)? A) He will have normal male external genitalia. B) He will have ovaries and female-appearing external genitalia but male secondary sex characteristics. C) He will have female-appearing external genitalia and ovaries, but internal structures derived from the Wolffian ducts. D) He will have male external genitalia but will have female-appearing internal structures derived from the Müllerian ducts. E) He will have undescended testes, but female external genitalia and secondary sex characteristics.

E

Which of the following is NOT a function of estrogen during pregnancy? A) Stimulation of myometrial growth B) Maintenance of the endometrium C) Stimulation of prolactin secretion D) Stimulation of breast development E) Controlling milk ejection

E

Which of the following is NOT a part of the swallowing reflex? A) Respiration is inhibited. B) The glottis closes. C) The upper esophageal sphincter relaxes. D) The lower esophageal sphincter relaxes. E) The pyloric sphincter relaxes.

E

Which of the following is NOT true of pituitary growth hormone? A) It directly promotes protein anabolism in many cells. B) It causes differentiation of precursor cells that then respond to IGF-I by proliferating. C) Hypersecretion of growth hormone in adults leads to acromegaly. D) It is absent or deficient in pituitary dwarfs. E) It is necessary for fetal growth.

E

Which of the following is TRUE of follicular estrogen synthesis? A) Granulosa cells synthesize an androgen which the theca cells convert to estrogen. B) Sertoli cells synthesize an androgen which the granulosa cells convert to dihydrotestosterone. C) Theca cells synthesize an androgen which luteal cells convert to estrogen. D) Production of progesterone by the corpus luteum requires cooperative interaction between two cell types. E) Theca cells synthesize an androgen which the granulosa cells convert to estrogen.

E

Which of the following is a major metabolic effect of glucagon? A) Increased glucose uptake by muscle cells B) Increased triglyceride synthesis in adipose tissue C) Decreased glycogenolysis in muscle D) Decreased gluconeogenesis in liver E) Increased glycogenolysis in liver

E

Which of the following most accurately describes the actions of aldosterone? A) Aldosterone increases Na+ secretion and K+ reabsorption in the cortical collecting ducts. B) Aldosterone increases Na+ reabsorption and K+ secretion in the proximal tubule. C) Aldosterone decreases Na+ reabsorption and K+ secretion in the cortical collecting ducts. D) Aldosterone increases Na+ secretion and K+ reabsorption in the proximal tubule. E) Aldosterone increases Na+ reabsorption and K+ secretion in the cortical collecting ducts.

E

Which of the following statements about neural control of digestion is FALSE? A) Parasympathetic stimulation is excitatory to digestion. B) Sympathetic stimulation is generally inhibitory to digestion. C) Local neural networks (nerve plexuses) regulate digestive functions. D) Nerve plexuses receive input from the autonomic nervous system E) Parasympathetic stimulation inhibits GI exocrine gland secretions.

E

Which of the following statements is FALSE regarding oxytocin? A) It is secreted by the posterior pituitary. B) It is regulated by a positive feedback loop. C) It facilitates the birth process. D) It stimulates uterine contractions. E) It stimulates synthesis of breast milk.

E

Which of the following statements is NOT true of the endocrine system? A) It is one of two major regulatory systems of the body. B) It is composed of glands that secrete chemical messengers into the blood. C) It is an important regulator of homeostatic mechanisms. D) It influences and is influenced by the nervous system. E) Most of its components are anatomically connected, like most other systems of the body.

E

Which of the following would cause a decrease in the excretion of Na+ and water? A) Increased hydrostatic pressure in the afferent renal arterioles B) Increased mean arterial blood pressure C) Increased atrial distension D) Decreased aldosterone secretion E) Decreased atrial natriuretic peptide secretion

E

Which of the following would lead to an increase in the secretion of gastrin? A) Adding chyme to the duodenum B) An increase in the presence of starch in the lumen of the stomach C) An increase in the presence of lipids in the duodenum D) An increase in the presence of peptides in the duodenum E) An increase in the presence of peptides in the stomach

E

Which of these are secretions of the parietal cells of the gastric mucosa? A) HCl and pepsinogen B) pepsinogen and intrinsic factor C) gastrin and intrinsic factor D) HCl and gastrin E) HCl and intrinsic factor

E

Which of these could be a cause of hypertrophy of the adrenal cortex? A) Autoimmune destruction of receptors for adrenocorticotropic hormone (ACTH) B) Autoimmune destruction of hypothalamic cells that secrete corticotropin-releasing hormone C) Injection of an excess of cortisol D) Destruction of the anterior pituitary by a stroke E) Low levels of cortisol in the circulation

E

Which of these is a function of intrinsic factor? A) It is secreted by salivary glands, and is necessary for digestion of vitamin B12. B) It is secreted by parietal cells in the gastric mucosa, and its main function is causing insulin release. C) It is secreted in the stomach, and its main function is activating pepsinogen into pepsin. D) It is secreted by the small intestine mucosa, and its main function is initiating the intrinsic electrical activity of the pacemaker cells in the stomach. E) It is secreted in the stomach, and a deficiency of it would cause anemia.

E

Which one of the following substances is LEAST dependent on the kidney for regulation of its plasma concentration? A) Water B) Na+ C) K+ D) HPO42- E) Glucose

E

Which would NOT be caused by a decrease in plasma Ca2+ levels in an otherwise normal person? A) An increase in plasma parathyroid hormone levels B) An increase in plasma 1,25-dihydroxyvitamin D3 levels C) An increase in nerve and muscle excitability D) A decrease in the filtered load of Ca2+ E) An increase in bone density

E

Which would most directly decrease the production of aldosterone? A) Going on a low-salt diet B) Ingesting extra dietary vitamin D C) Reducing dietary tyrosine D) Increasing dietary cholesterol E) Injecting a drug that blocks the production of angiotensin II

E

You are designing an experiment to quantify the levels of stress experienced by different individuals. Which of the following tests would reveal insight as to a patient's chronic stress? A) Testing plasma levels of growth hormone B) Testing plasma levels of parathyroid hormone C) Testing plasma levels of dopamine D) Testing plasma levels of follicle-stimulating hormone E) Testing plasma levels of cortisol

E

1. Which of the following statements regarding the structure and function of cell membranes is correct? A. The phospholipid bilayer is arranged so that the hydrophilic heads of the phospholipid molecule face the extra- and intracellular fluids. B. Some proteins integrated into the membrane serve as channels for the passage of nonpolar molecules through the membrane. C. Peripheral membrane proteins function as channels associated with transport of ions through the membrane. D. Cholesterol molecules function to transport ions from one side of the plasma membrane to the other. E. The plasma membrane is a layer of phospholipid molecules with their hyrophilic head groups in contact with the extracellular solution and the hydrophobic tail groups in contact with the intracellular solution.

a

11. Cells can increase their responsiveness to an external chemical regulator by A. increasing the number of their transmembrane receptors by exocytosis. B. decreasing the number of their transmembrane receptors by endocytosis. C. uncoupling their receptors from the second message generator. D. increasing the number of their transmembrane receptors by endocytosis. E. mutating their extracellular receptors so that the affinity for the chemical regulator is reduced.

a

11. Which is true about mediated transport of substances across cell membranes? A. It involves a specific membrane protein that functions as a carrier molecule. B. It always involves the movement of substances against a concentration gradient. C. It is always directly coupled to the splitting of ATP molecules. D. There is no limit to how fast it can occur as the concentration gradient becomes larger. E. It is the main mechanism for transporting hydrophobic molecules across membranes.

a

12. The diffusion potential due to the concentration gradient for Na+ across a nerve cell membrane: A. favors its movement into the cell at the resting membrane potential. B. favors its movement out of the cell at the resting membrane potential. C. is equal and opposite to the electrical potential acting on Na+ at the resting membrane potential. D. Is in the same direction as the diffusion potential due to the concentration gradient for K+. E. favors movement of Na+ in the opposite direction as the electrical potential acting on Na+ at the resting membrane potential.

a

13. What term is used to describe the steady state value for any variable that the body attempts to maintain? A. Set point B. Equilibrium potential C. Error signal D. Reflex arc E. Median value

a

13. Which is true regarding the mediated transport of a substance across a plasma membrane? A. It depends upon the binding of that substance to a specific site on the membrane protein. B. It depends upon movement of proteins from one side of the membrane to the other. C. It always increases in direct proportion to the increasing concentration of the substance on one side of the membrane. D. Both "depends upon the binding of that substance to a specific site on the membrane protein" and "depends upon movement of proteins from one side of the membrane to the other" are correct. E. All of the choices are correct.

a

13. Which of the following is not true of the cytoskeleton? A. It refers to the cellular components of bone. B. It refers to a network of cytoplasmic filaments. C. It is important for cellular movement. D. It helps to determine a cell's shape. E. It includes actin filaments.

a

13. Which would result from an increase in the extracellular concentration of K+ above normal? A. depolarization of resting nerve cells B. hyperpolarization of resting nerve cells C. The potassium equilibrium potential of nerve cells would become more negative. D. The sodium equilibrium potential would become less positive.

a

14. Cell A is a gland cell that makes and secretes proteins. Cell B is another type of cell that synthesizes and secretes steroids. If you were to look at electron micrographs of the two cells, what differences would you expect to see? A. Cell A would have more granular endoplasmic reticulum than cell B. B. Cell A would have fewer secretion granules than cell B. C. Cell A would have more agranular endoplasmic reticulum than cell B. D. Cell B would have more granular endoplasmic than cell A. E. The two cells would most likely look identical.

a

14. Which is TRUE about the Na+, K+ ATPase pump in neurons? A. It generates a small electrical potential such that the inside is made negative with respect to the outside. B. It maintains a concentration gradient for K+ such that diffusion forces favor movement of K+ into the cell. C. It maintains an electrical gradient at the equilibrium potential of K+. D. It transports equal numbers of sodium and potassium ions with each pump cycle. E. It pumps 3 Na+ ions into the cell for every 2 K+ ions it pumps out.

a

14. Which of the following is NOT typically true about G proteins? A. They act as second messengers. B. They can be stimulatory for second-messenger production. C. They can be inhibitory for second-messenger production. D. They can act as transducers for activated receptors by opening or closing ion channels.

a

15. Feedforward regulatory processes A. work in anticipation of changes in regulated variables. B. are identical to positive feedback processes C. Lead to instability of the regulated variable D. Maximize fluctuations in the regulated variable E. tend to force physiological variables away from their set point.

a

16. In Tay Sach's disease, lysosomes do not function normally. Predict which condition would be a symptom of Tay Sach's disease. A. There is excessive accumulation of damaged organelles in a cell. B. Cells are unable to synthesize proteins. C. Cells can only make ATP by glycolysis. D. Proteins cannot be secreted by exocytosis. E. Steroid hormones can't be produced.

a

17. In which organelle are carbohydrate chains added to glycoproteins? A. golgi apparatus B. smooth endoplasmic reticulum C. nucleus D. lysosome E. mitochondrion

a

17. The equilibrium potential of K+ ions in nerve cells is about -90 mV. The membrane potential of typical nerve cells at rest is -70 mV. Therefore A. Increasing the permeability of a resting neuronal membrane to K+ will make the membrane potential more negative inside with respect to outside. B. In resting neurons, there is a net diffusion of K+ into the cell. C. changing the resting membrane potential of a neuron to -80 mV would increase K+ diffusion rate out of the cell. D. potassium is the only permanent ion at rest. E. there must be another permanent ion with an equilibrium potential more negative than -90 mV.

a

17. Which of the following statements is true? A. Phosphorylation by protein kinases can stimulate or inhibit the activity of effector proteins. B. Ca2+ is not a second messenger. C. Phosphodiesterase converts GMP into cGMP D. Conversion of ATP to cAMP is a phosphorylation reaction. E. Phospholipase C converts ATP to cAMP.

a

18. Which of the following are known to be second messengers? A. diacylglycerol B. phospholipase C C. ATP D. adenylyl cyclase E. epinephrine

a

19. If the ATP-generating mechanisms in a cell are poisoned and the cell depletes its ATP reserves, which would occur first? A. Primary active transport of molecules would cease. B. Secondary active transport of molecules would cease. C. Facilitated diffusion of molecules would cease. D. Ion concentration gradients would reach equilibrium across the cell membrane. E. All transport processes would cease immediately when the ATP was depleted.

a

2. Which is TRUE regarding diffusion? A. It depends upon the random motion of molecules. B. It results in net movement of molecules from regions of low concentration to regions of high concentration. C. It is the principle mechanism by which molecules are moved over large distances in the body. D. It requires energy in the form of ATP. E. It requires integral membrane proteins to occur.

a

22. If all other conditions remain the same and the concentration of a nonpenetrating solute increases inside a cell, which is most likely to occur? A. Water will tend to enter the cell because the interior has an increased osmolarity. B. Water will tend to leave the cell because the interior has an increased osmolarity. C. Water will tend to enter the cell because the interior has a decreased osmolarity. D. Water will tend to leave the cell because the interior has a decreased osmolarity. E. The solute will diffuse across the membrane until its concentration is equal on both sides of the membrane.

a

23. What will happen if a normal cell is placed in a hypotonic solution? A. It will swell in size. B. It will shrink in size. C. It will stay the same size. D. The result can't be predicted.

a

25. How does the synthesis of proteins that are destined to be secreted differ from that of proteins with functions inside the cell? A. Proteins destined to be secreted have a signal sequence. B. Proteins destined to be secreted are synthesized on free ribosomes. C. Proteins destined to be secreted are packaged into secretory vesicles in the granular endoplasmic reticulum. D. Only proteins destined for secretion need transfer RNA to be synthesized. E. Genes for proteins with functions inside the cell are located in the cytoplasm.

a

25. If a normal cell is placed into an unknown solution and it shrinks, what can be concluded about the unknown solution? A. Its nonpenetrating solute concentration is greater than that of a normal cell. B. Its nonpenetrating solute concentration is less than that of a normal cell. C. Its nonpenetrating solute concentration is equal to that of a normal cell. D. Its penetrating solute concentration is less than that of a normal cell. E. Its penetrating solute concentration is greater than that of a normal cell.

a

25. Which is true about neuronal membrane electrical and concentration gradients at the peak of the action potential? A. The electrical gradient is in a direction that would tend to move K+ out of the cell. B. The concentration gradient for K+ is in a direction that would tend to move it into the cell. C. The concentration gradient for K+ greatly increases compared to at rest. D. The concentration gradient for Na+ is in a direction that would tend to move it out of the cell. E. The electrical gradient for Na+ is in a direction that would tend to move it into the cell.

a

27. Heart rate is increased by the release of epinephrine by the adrenal medulla into the bloodstream. This is an example of A. endocrine control B. exocrine control C. paracrine control D. direct neural control E. positive feedback

a

27. Which of these is an example of a hypertonic solution? A. 200 mM NaCl B. 300 mM glucose C. 100 mM MgCl2 D. 400 mM urea E. 100 mM NaCl

a

29. The tall slender body shape that helps to dissipate heat in people native to equatorial regions is an example of A. an adaptation. B. acclimatization. C. set point resetting. D. homeostasis. E. phase-shift.

a

3. The study of disease states in the body is called A. Pathophysiology B. Anatomy C. Homeostasis D. Biology E. Histology

a

31. Neuronal axons typically have abundant A. voltage-gated channels for Na+ that open in response to depolarization. B. voltage-gated channels for K+ that open in response to hyperpolarization. C. receptor-mediated channels for Na+. D. receptor-mediated channels for K+. E. voltage-gated channels for Ca2+.

a

31. Two receptors, A and B, can bind the same chemical signaling molecules but the concentration of those molecules required to reach 50 percent saturation is twice as great for receptor B. Therefore the receptors have the same ________, but different ________. A. specificity, affinity B. specificity, saturation C. affinity, specificity D. affinity, competition E. competition, affinity

a

32. What is the location of the internal pacemaker that sets biological rhythms? A. suprachiasmatic nucleus of the brain B. ventricles of the heart C. endocrine gland in the gonads D. photoreceptors of the eye E. the adrenal glands

a

32. Which of the following statements regarding action potentials generated in a neuronal membrane is not true? A. Action potentials travel decrementally down the membrane. B. An action potential generates a new action potential in an adjacent area of membrane. C. An action potential generates a local current that depolarizes adjacent membrane to threshold potential. D. Action potentials are usually initiated at the initial segment of a neuron. E. An action potential generated by a threshold stimulus is the same size as one generated by a supra-threshold stimulus.

a

33. A protein is found in blood that is produced by the pancreas and acts on receptors of cells in the liver. What type of physiological regulator is it most likely to be? A. a hormone B. an autocrine signal C. a paracrine signal D. a neurotransmitter E. an enzyme

a

33. Which of the following statements concerning the properties of action potentials is true? A. The rate of propagation of an action potential down an axon is independent of stimulus strength. B. Action potentials can undergo summation. C. A supra-threshold stimulus can stimulate an action potential during the absolute refractory period. D. Action potentials generally propagate from the axon terminal toward the initial segment. E. Increasing the size of a stimulus will increase the amplitude of an action potential.

a

35. At any given time it is possible to see endocytotic vesicles docked to intracellular organelles such as those which may receive their contents for modification, synthesis, packaging, and then release. Which organelle would be most likely to be involved in this process as a destination for a transport vesicle formed for this purpose along the plasma membrane? A. Golgi apparatus B. peroxisome C. lysosome D. mitochondrion E. ribosome

a

36. Eating a bag of salty potato chips without increasing sodium excretion would result in what state? A. positive sodium balance B. negative sodium balance C. stable sodium balance D. It can't be determined without knowing the size of the sodium pool

a

38. What is the most common transport mechanism by which glucose and other organic solutes cross the luminal membrane of an epithelial cell layer? A. secondary active cotransport with Na+ B. glucose ATPase pump C. facilitated diffusion D. simple diffusion through the paracellular pathway E. through an ion channel

a

4. Which of the following statements regarding the diffusion of a nonpolar solute X across a cell membrane is correct? A. At equilibrium the net flux of X across the cell membrane is zero. B. At equilibrium, X will cease moving across the cell membrane. C. Solute X will be unable to reach equilibrium because it is nonpolar. D. Solute X will reach equilibrium, but the concentration will be much higher on the inside of the cell because of the smaller volume. E. At equilibrium, movement of X across the membrane will be much faster in one direction than in the other.

a

4. Which of these is a lipid soluble messenger? A. thyroid hormone B. protein kinase C. glucose D. sodium E. cyclic AMP

a

41. At an excitatory chemical synapse between two neurons, A. there is increased permeability of the postsynaptic cell to both Na+ and K+. B. a small hyperpolarization of the postsynaptic membrane occurs when the synapse is activated. C. an action potential in the presynaptic neuron always causes an action potential in the postsynaptic neuron. D. excitation occurs because K+ enters the postsynaptic cell. E. action potentials spread through gap junctions between cells.

a

42. When a reversible chemical reaction is at equilibrium A. the rate of the reaction in one direction is equal to the rate in the reverse direction. B. the concentration of the reactants is equal to the concentration of the products. C. both the forward and reverse reactions stop occurring. D. the supply of reactants has become exhausted, so the forward reaction ceases and the rate of the reverse reaction increases. E. catalysts no longer influence either the forward or reverse reaction rates.

a

44. An EPSP: A. is a direct result of the opening of ligand-gated channels permeable to both Na+ and K+ ions. B. is a direct result of the opening of voltage-gated channels permeable to both Na+ and K+ ions. C. stabilizes the membrane to remain at its resting potential. D. opens voltage-gated Ca2+ channels in the presynaptic membrane. E. occurs when voltage-gated Cl- channels open in a postsynaptic cell membrane.

a

45. At equilibrium, in an irreversible reaction: A. almost all of the reactants will have been converted to products. B. there will be an equal number of products and reactants. C. there will be very few molecules of product. D. small amounts of energy will have been released. E. large amounts of energy must have been input to reach that state.

a

45. Which best describes temporal summation? A. A synapse is stimulated a second time before the effect of a first stimulus at the synapse has terminated. B. It only refers to addition of EPSPs. C. Two synapses on different regions of a cell are stimulated at the same time. D. It always brings a postsynaptic cell to threshold. E. The size of an EPSP depends on the size of the stimulus.

a

46. What is the function of a catalyst in a chemical reaction? A. It lowers the activation energy of the reaction. B. It is split into individual molecules that are products of the reaction. C. It changes the energy content of the reactants. D. It generates heat that makes the reaction proceed. E. It ensures that the reaction can proceed only in one direction, from reactants to products.

a

47. Enzymes A. are catalysts in chemical reactions. B. can be carbohydrate molecules. C. are broken down during chemical reactions they catalyze. D. have names generally ending in the suffix "-ose." E. are polymers of ATP that store large amounts of energy.

a

49. What is a coenzyme? A. an organic cofactor that directly participates as one of the substrates in an enzymatic reaction B. a metal, such as zinc, that participates in enzymatic reactions C. a term regarding enzymatic reactions that is synonymous with "cofactor" D. any ligand that binds to a specific site on a protein receptor E. an inorganic molecule that catalyzes enzymatic reactions

a

52. Which of the following statements about acetylcholine is correct? A. Acetylcholine binds to nicotinic and muscarinic receptors. B. Acetylcholine binds to adrenergic receptors. C. Acetylcholine synthesis is catalyzed by acetylcholinesterase D. Acetylcholine is the neurotransmitter released by most sympathetic postganglionic neurons. E. Acetylcholine is generally transported back into presynaptic nerve terminals without being enzymatically degraded.

a

53. Which of the following statements regarding neurotransmitters is TRUE? A. Acetylcholine is broken down by enzymes present on postsynaptic cell membranes. B. Acetylcholine that is released at synapses binds to adrenergic receptors in the postsynaptic cell membrane. C. Catecholamines are the most abundant neurotransmitters in the central nervous system. D. Opiate drugs, such as morphine, are antagonists of a class of neurotransmitters called endorphins. E. Gamma-aminobutyric acid (GABA) is a major excitatory transmitter in the central nervous system.

a

54. ATP is A. a nucleotide. B. a lipid. C. an amino acid. D. a carbohydrate. E. a steroid.

a

56. Which of the following enzymes is important for the metabolism of catecholamines? A. Tyrosine hydroxylase B. Acetylcholinesterase C. Superoxide dismutase D. Hydrocortinase E. Adenyl cyclase

a

56. Which of the following metabolic pathways can proceed in the absence of oxygen? A. glycolysis B. oxidative phosphorylation C. Krebs cycle D. the breakdown of fatty acids to CO2 and H2O E. None of the choices are correct.

a

58. Serotonin: A. acts as a neuromodulator. B. is a catecholamine neurotransmitter. C. is a neuropeptide. D. has an inhibitory effect on pathways that are involved in the control of muscles. E. has an excitatory effect on pathways that mediate sensations.

a

58. Which of the following is NOT a product of glycolysis? A. carbon dioxide B. ATP C. pyruvate D. NADH

a

6. The difference in electrical charge between two points: A. is called the potential difference between those points. B. is called the diffusion potential between those points. C. is called the the current, and is expressed in the units of millimoles. D. is the same for all ions.

a

61. Which is one of the most abundant excitatory neurotransmittors in the CNS? A. Glutamate B. Dopamine C. Norepinephrine D. Gamma-aminobutyric acid (GABA) E. Endorphin

a

65. Electrons transferred along the electron transport chain can be used to form: A. water. B. glucose. C. ATP. D. NADH. E. acetyl coenzyme A.

a

66. Which of the following does NOT result from the binding of nicotine with nicotinic acetylcholine receptors? A. a mild form of skeletal muscle paralysis that creates a more relaxed state B. signal transmission at neuromuscular junctions C. generation of excitatory signals within autonomic ganglia D. the release of norepinephrine, dopamine, and epinephrine E. facilitation of the release of multiple neurotransmitters within the brain, including the "reward pathway" involving dopamine

a

7. Methadone is a drug given to treat heroin addicts. It works by binding to the same receptors as heroin but with greater affinity, thereby preventing heroin from binding. This is an example of A. competition. B. down regulation. C. signal transduction. D. agonistic behavior. E. up regulation.

a

70. The cerebellum: A. is important for coordinating body movement. B. is the gray matter covering the entire surface of the brain. C. is the same thing as the brain stem. D. is in the anterior portion of the brain, just above the eyes. E. is one of the basal nuclei that is found deep inside the cerebrum.

a

72. Which is a FALSE statement about the cerebrospinal fluid? A. It has the same composition as blood plasma. B. It acts as a cushion for the brain and spinal cord. C. It is secreted by cells lining the ventricles of the brain. D. It circulates within and between brain ventricles and surrounds the spinal cord. E. It is in diffusion equilibrium with the extracellular fluid of the central nervous system.

a

73. Which of the following is NOT characteristic of the sympathetic division of the ANS? A. Preganglionic neurons tend to be long, with the ganglion located in or near the effector target tissue(s). B. Preganglionic neurons are short and synapse in ganglia located near the spinal cord. C. Preganglionic neurons release acetylcholine at synapses with postganglionic neurons. D. Postganglionic neurons release norepinephrine at their neuroeffector junctions. E. Postganglionic neuron cell bodies are located in the collateral chain ganglia (sympathetic trunk).

a

8. What cellular structures are most closely associated with vaults? A. nuclear pores B. peroxisomes C. lysosomes D. mitochondria E. gap junctions

a

9. Which is TRUE about the resting membrane potential? A. It requires very few ions to be distributed unevenly. B. It has the same value in all cells. C. It is oriented so that the cell's interior is positive with respect to the extracellular fluid. D. Only nerve and muscle cells have a potential difference across the membrane at rest. E. It is not altered by changing concentration gradients of permeating ions.

a

1. What are the most common receptors for hydrophilic intercellular messenger molecules? A. peripheral membrane proteins B. integral membrane proteins C. specialized phospholipids within the membrane D. nucleic acids E. intracellular proteins

b

1. Which of these is NOT one of the four general categories of cells that make up the human body? A. epithelial cells B. collagen cells C. connective tissue cell D. neuron E. muscle cell

b

10. Which concept is the defining feature of the discipline of physiology? A. descent with modification B. homeostasis C. evolution D. dimorphism E. differentiation

b

12. In which of these would an injected drug be the most effective agonist for an endogenous chemical messenger? A. The drug has a lower affinity for the messenger's receptors than the messenger does. B. The drug achieves 50 percent saturation of the messenger's receptors at a lower concentration than that required by the messenger. C. The drug does not couple to the binding site of the messenger's receptor. D. The drug binds to an alternate binding site on the protein receptor and reduces its affinity for the endogenous chemical messenger.

b

15. Which of these would occur if the concentration of ATP were depleted in a typical nerve cell? A. Resting membrane potential would become more negative. B. Resting membrane potential would become less negative. C. The concentration gradient for Na+ would remain the same. D. The resting membrane potential would eventually become positive inside with respect to outside. E. There would be no change in the resting membrane potential.

b

16. Second messengers: A. are necessary for all receptor signal transduction mechanisms. B. act in the cell cytoplasm. C. only function as intercellular messengers. D. always function to activate enzymes. E. are always proteins.

b

16. Which situation describes a feedforward mechanism? A. Blood glucose returns toward normal an hour after a meal. B. The smell of rotten food on a plate triggers the vomit reflex. C. A drop in core body temperature triggers shivering. D. An increase in core body temperature stimulates sweating E. Food in the stomach triggers the production of stomach acid.

b

19. If the amount of sodium in the blood decreases, what would a negative feedback control mechanism be expected to do? A. Decrease the amount of sodium in the blood. B. Increase the amount of sodium in the blood. C. Leave the amount of sodium unchanged. D. Change the set point for sodium. E. Inhibit the ingestion of more sodium.

b

2. Physiology is the study of A. How two organisms interact B. How organisms function C. The spread of diseases D. The structure of the body

b

20. An action potential in a neuronal membrane differs from a graded potential in that: A. an action potential requires the opening of Ca2+ channels, whereas a graded potential does not. B. an action potential is propagated without decrement, whereas a graded potential decrements with distance. C. an action potential has a threshold, whereas a graded potential is an all-or-none phenomenon. D. movement of Na+ and K+ across cell membranes mediate action potentials, while graded potentials do not involve movement of Na+ and K+. E. action potentials vary in size with the size of a stimulus, while graded potentials do not.

b

20. What is a codon? A. a triplet of deoxyribonucleotides B. a triplet of ribonucleotides C. a sequence of ribonucleotides on tRNA D. a very small gene E. a very small genome

b

22. Which must happen in order for an action potential to begin? A. The membrane potential must be at the Na+ equilibrium potential. B. Na+ influx must exceed K+ efflux. C. The membrane must be out of the relative refractory period. D. Na+ channels must all be inactivated. E. Multiple inhibitory postsynaptic potentials (IPSPs) must summate.

b

23. The hormone insulin enhances the transport of glucose into body cells. Its secretion is controlled by a negative feedback system between the concentration of glucose in the blood and the cells that secrete insulin. Which of the following statements is most likely to be correct? A. A decrease in blood glucose concentration will stimulate insulin secretion, which will in turn lower the blood glucose concentration still further B. An increase in blood glucose concentration will stimulate insulin secretion, which will in turn lower the blood glucose concentration C. A decrease in blood glucose concentration will stimulate insulin secretion, which will in turn increase the blood glucose concentration D. An increase in blood glucose concentration will stimulate insulin secretion, which will in turn increase the blood glucose concentration still further

b

23. What is the term for the segments of primary RNA that are cleaved and discarded by spliceosomes? A. codons B. introns C. exons D. anticodons E. genes

b

23. Which describes the response of the voltage-gated channels when an axon is stimulated to threshold? A. K+ channels open before the Na+ channels. B. Na+ channels are activated and then inactivated. C. K+ channels open at the same time as the Na+ channels. D. K+ channels are opened when Na+ binds to the channel. E. K+ influx causes Na+ channels to inactivate.

b

24. During the rising (depolarizing) phase of a neuronal action potential, A. PK+ becomes much greater than PNa+. B. PNa+ becomes much greater than PK+. C. PK+ is the same as PNa+. D. Na+ efflux (flow out of the cell) occurs. E. K+ flows rapidly into the cell.

b

24. How are endocrine glands and hormones involved in homeostatic reflexes? A. Endocrine glands can be receptors, and hormones can be effectors. B. Endocrine glands can be integrators and hormones can be efferent pathways. C. Endocrine glands can be efferent pathways and hormones can be effectors D. Endocrine glands are not part of reflex mechanisms, but hormones can be afferent or efferent pathways E. They are not involved; reflexes only involve actions of the nervous system.

b

27. Which is the best definition of the term, ligand? A. a receptor composed of protein B. a molecule or ion that binds to a protein C. any molecule found in the intracellular fluid D. any molecule found in the extracellular fluid E. a carbohydrate molecule that binds weakly to membrane phospholipids

b

28. Which of the following solutions is not isotonic to human cells? A. 150 mM NaCl B. 300 mM urea C. 100 mM MgCl2 D. 300 mOsm NaCl

b

28. Which of the following statements about the phases of a neuronal action potential is true? A. During the after-hyperpolarization phase, the permeability of the membrane to sodium ions is greater than its permeability to potassium ions. B. During the after-hyperpolarization phase, the permeability of the membrane to potassium ions is greater than its permeability at rest. C. During the repolarizing phase, the permeability of the membrane to sodium ions is greater than its permeability to potassium ions. D. Potassium channels inactivate during the depolarization phase. E. Repolarizing to negative membrane potentials causes the sodium channels to inactivate.

b

3. A solute, X, is placed in compartment A of a two compartment container and allowed to diffuse to compartment B and attain diffusion equilibrium. At that point in time, A. there will be no further movement of any solute molecules between compartments. B. solutes will be moving in both directions equally. C. solutes will only continue to move from A to B. D. solutes will only continue to move from B to A. E. the concentration in compartment B will be much higher than that in compartment A.

b

3. The cell membrane is composed mainly of: A. cytosol. B. phospholipids and proteins. C. phospholipids and nucleic acids. D. water. E. proteins and glycocalyx.

b

30. Which of the following statements regarding endocytosis and exocytosis is correct? A. Endocytosis is a method by which large molecules may be secreted from a cell. B. Exocytosis is a method by which large molecules may be secreted from a cell. C. Pinocytosis is a form of endocytosis; phagocytosis is a type of exocytosis. D. Pinocytosis is a form of exocytosis; phagocytosis is a type of endocytosis. E. Pinocytosis and phagocytosis are both types of exocytosis.

b

32. What type of cellular transport involves the cytosolic protein clathrin? A. exocytosis B. receptor-mediated endocytosis C. primary active transport D. secondary active transport E. pinocytosis

b

32. Which describes the action of a ligand's competitive antagonist? A. It decreases the amount of ligand required to activate the protein. B. It blocks the binding of the ligand to its binding site and prevents activation of the protein. C. It binds to a different binding site on the protein, altering the protein shape so that a ligand cannot bind. D. When present, it increases the binding of a ligand to its receptor.

b

34. How is the strength of a stimulus encoded by neurons? A. by the size of action potentials B. by the frequency of action potentials C. by the duration of action potentials D. by whether the action potential peak is positive or negative

b

35. A burn patient ingests 100 grams of protein per day and loses 110 grams of protein per day due to the injury. What is the overall protein state of the patient? A. Positive protein balance B. Negative protein balance C. Stable protein balance D. A state that can't be determined

b

36. In skeletal muscle, when calcium binds to the regulatory protein troponin, it increases the affinity of neighboring troponin molecules for calcium. This is an example of A. allosteric modulation. B. cooperativity. C. covalent modulation. D. phosphorylation. E. saturation.

b

37. The regions of axon membrane that lie between regions of myelin are the A. islets of Langerhans. B. nodes of Ranvier. C. synaptic membranes. D. glial cells. E. dens of iniquities.

b

37. Which lists the epithelial cell barrier compartments in the order of the typical Na+ concentrations, from highest to lowest? A. intracellular, lumen side, blood side B. blood side, lumen side, intracellular C. blood side, intracellular, lumen side D. lumen side, intracellular, blood side E. lumen side, blood side, intracellular

b

4. Which is NOT a connective tissue cell? A. bone cells B. skeletal muscle cells C. blood cells D. fat cells E. cartilage cells

b

40. How does a chemical catalyst increase the rate of a reaction? A. by acting as one of the reactant molecules B. by decreasing the energy of activation C. by increasing the energy content of the product molecules D. by increasing the temperature of a solution E. by phosphorylating a reactant

b

41. With regard to reversible chemical reaction rates, which of the following statements is true? A. Decreasing the concentration of the reactants will increase the forward reaction rate. B. Increasing the temperature will increase the forward and reverse reaction rate. C. Increasing the activation energy will increase the forward reaction rate. D. Increasing the concentration of the products will increase the forward reaction rate. E. As a reaction progresses, the rate of the forward reaction increases as the concentration of reactants decreases.

b

42. An inhibitory postsynaptic potential: A. is produced by simultaneous increases in permeability to both Na+ and K+. B. occurs when a ligand-gated ion channel increases its permeability to K+. C. is a small depolarization in a postsynaptic cell. D. can be summed with other IPSPs to trigger an action potential in the postsynaptic cell. E. is produced by an increase in permeability to only Na+.

b

43. Which would be true about the following reaction? M + N Y + Z A. Adding a catalyst would alter the final concentrations of products and reactants at equilibrium. B. Starting at chemical equilibrium, increasing the concentration of M will transiently increase the rate of formation of Y and Z. C. Starting at chemical equilibrium, decreasing the concentration of M will increase the concentration of Y and Z. D. Both the reaction is reversible and at chemical equilibrium, increasing the concentration of M will drive the reaction to the left are correct E. Because the reactants and products are different molecules, this is not a reversible reaction.

b

44. Consider the reaction: H2CO3 CO2 + H2O + 4 kcal/mol. Which of the following is TRUE? A. The reaction is anabolic and the energy content of the reactant is greater than that of the products. B. The reaction is catabolic and the energy content of the reactant is greater than that of the products. C. The reaction is anabolic and the energy content of the products is greater than that of the reactant. D. The reaction is catabolic and the energy content of the products is greater than that of the reactant. E. The reaction is catabolic and the energy content of the products are equal to that of the reactant.

b

48. Which is TRUE about the initial segment of an axon? A. Its threshold potential is more positive than that of the cell body and dendrites. B. Its threshold potential is more negative than that of the cell body and dendrites. C. Synapses far from the initial segment are more effective in influencing whether an action potential will be generated in the axon than are synapses close to the initial segment. D. It is the region where neurotransmitter vesicles are docked and ready to be released by exocytosis. E. It can only conduct graded potentials because it lacks voltage-gated Na+ channels.

b

49. A presynaptic synapse: A. is a synapse between an axon terminal and a dendrite that can be either excitatory or inhibitory. B. is a synapse between an axon terminal and another axon's terminal that can be either excitatory or inhibitory. C. is any synapse onto a cell body, and they can be either stimulatory or inhibitory. D. is a synapse between an axon terminal and a dendrite of the same cell, which is always inhibitory. E. is a synapse between an axon terminal and another axon terminal that is always inhibitory.

b

5. What is the main function of cellular tight junctions? A. They resist forces that tend to pull cells apart. B. They form barriers that restrict the passage of materials through the extracellular space between cells. C. They are protein channels that allow the movement of ions between the cytosol of adjacent cells. D. They are spaces that allow movement of substances between the interior of the nucleus and the cytosol. E. They hold the genetic material in a tightly coiled conformation.

b

5. Which of the following is not true of glial cells? A. They form the myelin for axons. B. Neurons outnumber glial cells 10 to 1 in the nervous system. C. They deliver fuel molecules to neurons and remove the waste products of metabolism. D. They are important for the growth and development of the nervous system. E. They regulate the composition of the extracellular fluid in the CNS.

b

5. Which of the following would decrease the net flux of a penetrating solute into a cell? A. increasing the permeability constant for that solute B. decreasing the temperature C. increasing the concentration of the solute in the extracellular fluid D. increasing the area of the cell membrane E. decreasing the thickness of the membrane

b

50. Neuron X makes inhibitory axon-axon synaptic contact with neuron Y at the synapse of Y and neuron Z. Which will occur when action potentials are stimulated in neuron X? A. Neuron Y will be inhibited from reaching the threshold to fire an action potential. B. The release of neurotransmitter by neuron Y will be inhibited. C. The synapse between neurons Y and Z will be changed from an excitatory synapse to an inhibitory one. D. Neurons Y and Z will both be more likely to reach threshold and fire an action potential. E. Neurons Y and Z will both be less likely to reach threshold and fire an action potential.

b

52. An experimental subject is isolated in an underground room with no windows, no clocks, and no contact with the outside world. Researchers monitoring his behavior observe that he eats breakfast a little bit later each day. What term best describes the subject's biological activity? A. circadian rhythm B. free-running rhythm C. jet lag D. phase shift E. entrainment

b

59. The reactions of the Krebs cycle: A. take place in the cytosol of human cells. B. generate ATP directly by substrate-level phosphorylation. C. are important for the metabolism of carbohydrates but not other molecules. D. take place only when no oxygen is present. E. produce pyruvate and lactate as end products.

b

6. Cocaine lowers the levels of a chemical messenger in the brain called enkephalin. Researchers have found the number of enkephalin receptors to be higher in cocaine addicts than nonaddicted people. This is an example of A. saturation. B. up regulation. C. antagonism. D. affinity. E. down regulation.

b

65. Which of the following kinds of neurons is not generally cholinergic? A. somatic motor neurons B. postganglionic sympathetic neurons C. postganglionic parasympathetic neurons D. preganglionic sympathetic neurons E. preganglionic parasympathetic neurons

b

68. Synthesis of neuropeptides differs from that of other neurotransmitters because it: A. takes place in the axon terminals of neurons. B. takes place in the cell bodies of neurons. C. takes place on ribosomes in the postsynaptic cell's membrane. D. uses amino acids as precursor molecules. E. only takes place at synapses outside the central nervous system.

b

7. What is the term for the developmental process that leads to specialized cell types? A. genomics B. differentiation C. homeostasis D. positive feedback E. acclimatization

b

9. At very low concentrations, epinephrine causes an artery to dilate. At higher concentrations epinephrine causes the same artery to constrict. How can these different effects be explained? A. There is one type of epinephrine receptor that uses two second messenger systems. B. There are two types of epinephrine receptors with different affinities for epinephrine that use two different second messenger systems. C. There are two types of receptors for epinephrine that use the same second messenger system. D. At higher concentrations epinephrine can pass through the plasma membrane and directly stimulate contraction within the cell.

b

10. The organelles that digest engulfed bacteria and cell debris are the: A. peroxisomes. B. endosomes. C. lysosomes. D. ribosomes. E. mitochondria.

c

10. Which is TRUE about typical, resting neurons? A. The plasma membrane is most permeable to sodium ions. B. The concentration of sodium ion is greater inside the cell than outside. C. The permeability of the plasma membrane to potassium ions is much greater than its permeability to sodium ions. D. The plasma membrane is completely impermeable to sodium ions. E. The plasma membrane is completely impermeable to potassium ions.

c

10. Which is true about mediated transport across cell membranes? A. It refers to the movement of ions through protein channels. B. It refers both to simple diffusion and to the active transport of molecules. C. It is characterized by saturable carriers and a maximum velocity of transport. D. As the concentration gradient across a membrane increases, the transport rate always increases. E. It is nonspecific; any transporter can transport any molecule across the cell membrane.

c

10. With regard to the action of hormones and neurotransmitters on cellular receptors, which of these describes "amplification?" A. When the extracellular concentration of a chemical messenger reaches a very high level, it overwhelms transporter molecules and the chemical floods into the cell. B. Only hydrophilic first-messenger molecules can activate second messenger molecules within the cell cytosol. C. A single first messenger molecule activates multiple second messenger molecules, each of which activate thousands of enzymes. D. Some cellular receptors have such low affinity for chemical ligands that it can require a million or more molecules to activate them.

c

15. Neurons have a very prominent nucleolus which indicates that they are actively performing what function? A. making ATP B. synthesizing lipids C. manufacturing ribosomes D. breaking down carbohydrates E. dividing by mitosis

c

15. Which is NOT typically a step in the cAMP second-messenger system? A. A first-messenger binds to a transmembrane receptor. B. There is dissociation of G-protein subunits. C. An activated G-protein subunit phosphorylates cAMP-dependent protein kinase. D. Adenyl cyclase converts ATP into cAMP. E. Active cAMP-dependent protein kinase phosphorylates cell proteins.

c

16. What is the best description of a secondary active transport process that cotransports an amino acid with Na+? A. Both Na+ and the amino acid bind on the side of the membrane where the Na+ concentration is lowest, and undergo net movement to the other side of the membrane. B. Na+ and the amino acid bind to opposite sides of the membrane, and Na+ undergoes net transport from the side of the membrane with higher Na+ concentration toward the side with lower concentration, while the amino acid moves in the opposite direction. C. Both Na+ and the amino acid bind on the side of the membrane where the Na+ concentration is higher, and both are transported to the side of the membrane where the Na+ concentration is lower. D. Both Na+ and the amino acid bind to the side of the membrane where the amino acid concentration is highest and are transported toward the side of the membrane where the amino acid concentration is lower. E. Na+ and the amino acid bind to opposite sides of the membrane, and the amino acid undergoes net transport from the side of the membrane with lower Na+ concentration to the side of the membrane where the Na+ concentration is higher.

c

21. A threshold stimulus applied to an excitable membrane is one that is just sufficient to: A. trigger an excitatory postsynaptic potential. B. cause a change in membrane potential. C. trigger an action potential. D. be conducted to the axon hillock. E. depolarize a dendrite.

c

21. If pure water and a solution containing a nonpenetrating solute are separated by a membrane that is permeable only to water, what would occur? A. Water will diffuse by osmosis until the water concentrations in the two compartments become equal. B. Water will diffuse by osmosis until all of the the water is on the same side as the solute. C. Water will diffuse by osmosis toward the side with the solute, until stopped by opposing hydrostatic pressure. D. No movement will occur between the compartments. E. Water will diffuse by osmosis away from the side with the solute, until stopped by hydrostatic pressure.

c

21. Which one of the following is the correct sequence for a regulatory reflex arc? A. Stimulus, effector, efferent pathway, integrating center, afferent pathway, receptor B. Stimulus, receptor, efferent pathway, integrating center, afferent pathway, effector C. Stimulus, receptor, afferent pathway, integrating center, efferent pathway, effector D. Stimulus, effector, afferent pathway, integrating center, efferent pathway, receptor E. Effector, efferent pathway, integrating center, afferent pathway, receptor, stimulus

c

22. Identify the effectors in this homeostatic reflex: Eating a salt-rich meal increases blood volume and pressure, stretching blood vessel walls. Nerve signals sent to the brainstem stimulate changes in hormonal and neural signaling. The heart rate is slowed, blood vessel walls are relaxed, and the kidneys increase urinary salt. The blood pressure returns toward normal. A. brainstem and blood vessels B. blood vessels, hormones and nerves C. heart, kidneys and blood vessels D. brainstem, blood vessels, and kidneys E. hormones and nerves

c

22. RNA synthesis from a DNA template: A. is called translation of the message. B. requires DNA polymerase. C. is called transcription. D. occurs in the nucleolus. E. occurs in the cytoplasm.

c

22. What second messenger most directly causes calcium ions to be released from intracellular stores? A. diacylglycerol B. adenylyl cyclase C. inositol triphosphate D. phospholipase A E. phospholipase C

c

28. How is autocrine regulation best described? A. Chemical regulators are released directly into blood vessels. B. Chemical regulators released by cells affect the functional status of different kinds of cells in the vicinity of the secretory cell. C. Chemical regulators affect the same cells that produce them. D. Chemical regulators reach their site of action through a duct. E. Chemical regulators are continuously released in constant amounts by the cell.

c

29. A certain protein receptor is capable of binding the neurotransmitter epinephrine but does not bind to the neurotransmitters dopamine, glutamate, or serotonin. This is because the receptor displays what characteristic? A. saturation B. inhibition C. specificity D. acclimatization E. accommodation

c

29. Regarding the tonicity and osmolarity of solutions, which of the following statements is not true? A. The term "tonicity" refers to the effect that a solution has on the degree of stretch or shrinking of the cell membrane. B. The term "osmolarity" refers to the osmotic properties of a solution, regardless of its tonicity. C. Isotonic solutions are always isoosmotic. D. Hypoosmotic solutions are always hypotonic. E. Hypertonic solutions are always hyperosmotic.

c

3. Which of the following is not true about axon transport? A. It refers to the passage of materials from the cell body of a neuron to the axon terminals. B. It refers to the passage of materials from axon terminals to the cell body of a neuron. C. It refers to the transport of materials from the inside to the outside across the axonal membrane. D. It is especially important for maintaining the integrity of neurons with long axons.

c

30. After spending several days at a high altitude, where oxygen pressure is low, a person will begin to produce more red blood cells, which enhances the ability of blood to carry oxygen to the tissues. What term best describes this type of response? A. developmental acclimatization B. positive feedback C. physiological acclimatization D. feedforward regulation E. evolution

c

34. Once formed inside of the cell, most endocytic vesicles will fuse with which organelle for sorting? A. endoplasmic reticulum B. golgi apparatus C. endosome D. nucleus E. mitochondria

c

34. What is "allosteric modulation?" A. regulation of physiological functions by the sympathetic division of the autonomic nervous system B. modification of the functional state of a protein by temperature or pH C. the change in the shape and functional state of a protein that occurs when a ligand binds to a regulatory site D. the fact that all binding sites on a protein must be occupied to have a biological effect E. the ability of a single ligand-binding site to bind to molecularly similar ligands

c

35. Which of the following statements concerning the rate of action potential propagation is true? A. It is faster in small-diameter axons than in large-diameter axons. B. It is faster for a strong stimulus than for a weak one. C. It is faster in myelinated axons than in nonmyelinated axons. D. It is faster in the dendrites than in the axon. E. It occurs at the same rate in all axons, regardless of their diameter.

c

36. Which of the following statements about epithelial cells and epithelial transport is NOT correct? A. Epithelial cell membranes express different transport proteins on different cell surfaces. B. Epithelial cells lining the small intestine have Na, K-ATPase pumps only in their basolateral membranes. C. The plasma membrane of epithelial cells that faces the inside of a hollow or fluid-filled chamber in the body is called the basolateral membrane. D. The pathway taken by substances that flow between epithelial cells is called the paracellular pathway. E. When substances cross epithelial barriers by going through the cell membranes and cytosol, it is called the transcellular pathway.

c

39. Exocytosis of neurotransmitter into the synaptic cleft is triggered by an influx of ______ in response to the arrival of an action potential in the axon terminal. A. K+ B. Na+ C. Ca2+ D. ATP E. Cl-

c

40. The main role of calcium ions at chemical synapses is to A. depolarize the axon terminal of the presynaptic cell. B. bind to neurotransmitter receptors on the postsynaptic cell. C. cause fusion of synaptic vesicles with the plasma membrane of the axon terminal. D. interfere with IPSPs in the postsynaptic cell. E. diffuse across the synaptic space and enter the postsynaptic cell.

c

46. A postsynaptic neuron has three presynaptic inputs - from neurons X, Y, and Z. Stimulation of neuron X causes the postsynaptic neuron to depolarize by 0.5 mV. When X and Y are stimulated simultaneously, the postsynaptic neuron depolarizes by 1 mV. When X and Z are stimulated simultaneously, however, there is no change in the membrane potential of the postsynaptic neuron. What is most likely true about presynaptic neurons Y and Z? A. They are both excitatory. B. They are both inhibitory. C. Y is excitatory and Z is inhibitory. D. Z is excitatory and Y is inhibitory.

c

47. A postsynaptic neuron has three presynaptic inputs - from neurons X, Y, and Z. When X and Y are stimulated simultaneously and repeatedly, the postsynaptic neuron reaches threshold and undergoes an action potential. When X and Z are stimulated simultaneously, however, there is no change in the membrane potential of the postsynaptic neuron. The simultaneous stimulation of X and Y is an example of A. temporal summation. B. presynaptic inhibition. C. spatial summation. D. neuronal divergence. E. presynaptic facilitation.

c

5. The process whereby repeated exposure to a hormone can cause a decrease in the number of receptors for that hormone is called A. competition. B. inhibition. C. down regulation. D. antagonism. E. saturation.

c

5. What is the principal function performed by epithelial cells? A. fat storage B. anchoring body structures C. forming boundaries between body compartments D. generating movement E. transmitting electrical signals

c

51. In order to increase the overall rate of a metabolic pathway consisting of five reactions catalyzed by five different enzymes, which must occur? A. increasing the reaction rate of all five enzymes B. increasing the reaction rate of the fastest enzyme C. increasing the reaction rate of the slowest enzyme D. increasing the reaction rate of the first enzyme in the pathway E. increasing the reaction rate of the last enzyme in the pathway

c

51. Which of the following is not known to be an important neurotransmitter in the CNS? A. Dopamine B. Acetylcholine C. Morphine D. Glutamate E. Substance P

c

52. ATP is A. formed during the hydrolysis of ADP. B. used by cells for the storage of energy. C. used to transfer energy within a cell. D. a protein. E. a molecule with less total energy content than ADP.

c

53. Which equation is most accurate? A. extracellular fluid volume + interstitial fluid volume = whole body fluid volume B. intracellular fluid volume + interstitial fluid volume = extracellular fluid volume C. extracellular fluid volume - interstitial fluid volume = plasma volume D. plasma volume + intracellular fluid volume = extracellular fluid volume E. total body fluid volume - intracellular fluid volume = interstitial fluid volume

c

55. Which is TRUE about glycolysis? A. It does not occur in the absence ofO2. B. It does not occur in the presence of O2. C. It may result in the formation of two molecules of lactate for each molecule of glucose. D. It is a series of metabolic reactions that occur mainly inside the inner membrane of a mitochondrion. E. It is a metabolic reaction in which glucose is manufactured from large carbohydrates like glycogen.

c

57. Nicotine is: A. a cholinergic antagonist. B. a beta-adrenergic agonist. C. a cholinergic agonist. D. an alpha-adrenergic antagonist. E. a neurotransmitter.

c

59. Alzheimer's disease is thought to involve primarily: A. loss of neurons that secrete or respond to catecholamines. B. loss of adrenergic neurons. C. loss of cholinergic neurons. D. loss of neurons that secrete or respond to dopamine. E. tumors that produce excess serotonin.

c

6. Equal amounts of two solutes, A and B, are placed into the same beaker of water at the same time. Solute A reaches diffusion equilibrium faster than solute B. What is the most likely explanation for this observation? A. The temperature was greater for solute A than solute B. B. The concentration gradient for A was greater than B. C. Solute A is smaller than solute B. D. Solute A traveled a shorter distance than solute B. E. Solute B is more soluble in water than solute A.

c

60. Which is a series of reactions by which fatty acid catabolism occurs? A. glycolysis B. lipogenesis C. beta-oxidation D. glycogenolysis E. phosphorylation

c

62. Most energy in the body is stored in what form? A. ATP B. glucose C. fat D. protein E. DNA

c

63. Ammonia is A. a waste product of fatty acid metabolism. B. exhaled by the lungs into the air. C. processed by the liver to form urea, a less toxic compound. D. a waste product of fatty acid metabolism and is excreted by the kidneys into urine. E. a polypeptide containing many amino acids.

c

67. Drug X interferes with the action of norepinephrine at synapses. Which of the following mechanisms would not explain the effects of X? A. X inhibits synthesis of norepinephrine at the axon terminal. B. X inhibits norepinephrine release from the terminal. C. X blocks reuptake of norepinephrine by the terminal. D. X is an adrenergic receptor antagonist. E. X stimulates the catabolism of norepinephrine.

c

7. According to the equation expressed as Ohm's law, which of these would cause the greatest increase in current? A. doubling both voltage and resistance B. reducing both voltage and resistance by half C. doubling voltage and reducing resistance by half D. reducing voltage by half and doubling resistance E. quadrupling both voltage and resistance

c

7. Protein channels that link the cytosol of adjacent cells are called ___________ junctions. A. cadherin B. tight C. gap D. conjunction E. nexus

c

71. The ________________ is best described as "an interconnected group of brain structures including parts of the frontal lobe-cortex, temporal lobe, thalamus, and hypothalamus, that is associated with learning, emotional experience, and behavior?" A. diencephalon B. cerebrum C. limbic system D. reticular formation E. cerebellum

c

8. The permeability of the plasma membrane to mineral ions: A. is not influenced by channels formed by proteins. B. is the same in all cell types. C. is affected by differences in electrical charge on the two sides of the membrane. D. is zero in all living cells. E. is only possible because of the hydrophobic interior of the lipid bilayer.

c

9. If a person begins to sweat upon entering a hot room but continued sweating is able to keep the body temperature constant, which of these best describes her condition? A. She is in an equilibrium state. B. She is not using energy to maintain a constant temperature. C. She is in a steady state D. She is using a positive feedback mechanism.

c

9. Ion channels in cell membranes: A. are nonspecific. B. are not affected by differences in electrical potential across the membrane. C. may open in response to binding a ligand. D. only allow ions to move from the extracellular fluid into the cell. E. only allow ions to move from the intracellular fluid out of the cell.

c

11. Describing a physiological variable as "homeostatic," means that it A. has varied from the normal value, and will remain constant at the new value. B. never varies from an exact set point value. C. is in an equilibrium state that requires no energy input to stay at the normal value. D. is in a state of dynamic constancy that is regulated to remain near a stable set point value. E. has no normal range, but will just change to match the outside environmental conditions.

d

11. The membrane potential of most neurons at rest is: A. equal to the equilibrium potential for potassium. B. equal to the equilibrium potential for sodium. C. slightly more negative than the equilibrium potential of potassium ion. D. more positive than the equilibrium potential for potassium. E. more positive than the equilibrium potential for sodium.

d

12. Which of the following situations best represents a homeostatic mechanism? A. A person who becomes very nervous begins to sweat profusely. B. After going outside on a hot day, the core body temperature increases. C. Increasing the size of fast food restaurant portions causes body weight to increase. D. After eating a large batch of salty popcorn, levels of salt in the urine increase. E. As age increases, the amount of calcium in bones tends to decrease.

d

13. A fat cell responds to the presence of the hormone epinephrine by increasing cytosolic cyclic AMP production, which leads to the catabolism of both glycogen and fat. What is the most likely explanation for this phenomenon? A. Epinephrine is binding to two types of receptors in the plasma membrane. B. The activated receptor complex stimulates production of two different second messengers. C. Cyclic AMP directly activates two kinds of enzymes. D. Cyclic-AMP-dependent protein kinase activates two kinds of enzymes.

d

14. Which is a form of cellular transport that involves the movement of molecules into cells without passing through the plasma membrane's structural matrix? A. diffusion B. osmosis C. facilitated diffusion D. endocytosis E. apoptosis

d

17. What is the general purpose of positive feedback mechanisms? A. to maintain a constant internal environment B. to anticipate changes in the environment C. to return a variable toward the set point D. to bring about a rapid change in the body E. to detect changes in the external environment

d

18. A cell is placed into a 1 millimolar solution of substance X and over time you witness the concentration of X inside the cell increase to 5 millimolar. What is the best explanation for this observation? A. X is moving into the cell by simple diffusion. B. X is moving into the cell by diffusion through a protein channel. C. X is moving into the cell by facilitated diffusion. D. X is moving into the cell by primary active transport. E. Water is leaving the cell by osmosis.

d

18. Which is true about the structure and function of a gene? A. It is an uncoiled protein that contains information necessary for the synthesis of other proteins. B. It is a sequence of nucleotides in DNA that acts as an enzyme to digest proteins. C. It is composed of many molecules of DNA and contains information needed to make RNA. D. It is a sequence of nucleotides in DNA that contains information necessary for the synthesis of proteins.

d

18. Which of the following statements concerning the permeability of a typical neuron membrane at rest is true? A. The permeability to Na+ is much greater than the permeability to K+. B. All of the K+ channels in the membrane are open. C. The voltage-gated Na+ channels are in the inactivated state. D. Most of the voltage-gated Na+ channels are in the closed state. E. There is equal permeability to Na+ and K+.

d

19. Amplification during a second messenger cascade is beneficial because amplification: A. takes small molecules and makes polymers out of them. B. results in the production of more of the first messenger. C. allows a cell to respond to more different hormones. D. allows small amounts of hormones to produce large responses in target cells.

d

2. Which is NOT a feature of the fluid-mosaic model of plasma membranes. A. Integral membrane proteins are embedded in the membrane. B. Phospholipids form a bilayer. C. Cholesterol associates with phospholipid molecules. D. Carbohydrates are linked to lipids and proteins on the interior surface, forming a glycocalyx layer. E. Peripheral membrane proteins associate with polar regions of integral membrane proteins.

d

2. Which is not true of myelin? A. It is a fatty membranous sheath. B. It is formed by glial cells. C. It influences the velocity of conduction of an electrical signal down an axon. D. It covers all parts of the neuron, including the axon, cell body, and dendrites.

d

20. "Osmosis" refers to the movement of what substance across semipermeable membranes? A. glucose B. charged particles C. lipid molecules D. water E. solutes

d

21. What codon corresponds to the DNA sequence G-T-A? A. G-T-A B. A-T-G C. C-A-T D. C-A-U E. T-A-C

d

24. What will happen if a normal cell is placed in a hyperosmotic solution? A. It will swell in size. B. It will shrink in size. C. It will stay the same size. D. It may swell, shrink, or stay the same size, depending upon the concentration of penetrating and nonpenetrating solutes in the solution.

d

24. Which is a function of transfer RNA (tRNA)? A. transport smRNA out of the nucleus B. binding specifically to mRNA and nonspecifically to amino acids C. binding specifically to amino acids and nonspecifically to mRNA D. binding specifically to both mRNA and to amino acids

d

26. Some neurons in the vagus nerve have synaptic connections to sinoatrial (pacemaker) cells in the heart. These neurons secrete acetylcholine, which ultimately results in a decreased heart rate. This is an example of A. endocrine control B. exocrine control C. hormonal control D. neural control E. paracrine control

d

26. Why do solutions for injection or infusion into people with low blood volume normally contain 150 mM NaCl? A. This is a hypotonic solution, which will cause water movement into dehydrated blood cells. B. This is an isotonic solution, and the water in it will follow Na and Cl into the the intracellular fluid compartment. C. This is a hypertonic solution, which will raise the blood volume and pressure more rapidly than an isotonic solution would. D. This is an isotonic solution, and NaCl will keep the added volume in the extracellular fluid compartment. E. NaCl are penetrating solutes, which will get them more quickly into blood cells to increase their volume.

d

27. Why are action potentials sometimes described as being "all-or-none" in character? A. The rate of propagation of an action potential down an axon is independent of stimulus strength. B. They are associated with an absolute refractory period. C. A supra-threshold stimulus is required to stimulate an action potential during the relative refractory period. D. An action potential occurs whenever a suprathreshold stimulus occurs, and its amplitude does not vary with the size of a stimulus. E. Action potentials are always the same size, even when ion gradients vary in size.

d

28. Which is NOT true about a binding site on a protein? A. It is an area of the protein with a shape complementary to that of a ligand. B. It is determined by the amino acid sequence of the protein. C. The binding of a ligand to a binding site typically changes the conformation of the protein. D. There can only be one binding site on a given protein. E. Binding of a ligand to the binding site typically activates or inactivates a protein's specific function.

d

30. The relative refractory period of an axon coincides with the period of A. activation and inactivation of voltage-dependent Na+ channels. B. Na+ permeability that is greater than that during the depolarization phase. C. increased K+ flux into the cell. D. increased K+ permeability of the cell. E. Increased Na+ flux through K+ channels.

d

31. Which form of endocytosis is nonspecific, in that it occurs by the formation of an invagination of the plasma membrane, which then fills with interstitial fluid of the immediate area? A. active transport B. hyperosmotic vesicular entrapment C. phagocytosis D. pinocytosis E. hydrosmosis

d

33. What is the defining characteristic of an allosteric protein? A. They contain two polypeptide chains, with each being an exact mirror image of the other. B. They contain only one ligand-binding site, but because the specificity is low, many different ligands can bind to it. C. They contain two ligand-binding sites, one that activates the protein when a ligand binds, and the other that inactivates the protein when the same ligand binds. D. They contain more than one ligand-binding site, and noncovalent binding of a ligand to one site alters the shape of other ligand binding sites. E. They contain no binding sites of their own, but act by modulating the activity of other proteins.

d

34. Which best describes how the total body balance of any chemical substance is determined? A. the rate the body produces the substance B. the rate the substance is secreted from the body C. the rate the substance is metabolized by the body D. the difference between the amount of substance lost from the body and the amount gained the body E. the amount produced by the body minus the amount metabolized by the body

d

35. Which of these covalently modifies proteins by phosphorylation? A. enzymes called protein phosphatases B. other proteins called allosteric proteins C. any ligand which can bind to specific binding sites D. enzymes called protein kinases E. metal ions, such as Fe2+

d

38. Which is FALSE about interneurons? A. They receive synaptic input from other other neurons in the CNS. B. They sum excitatory and inhibitory synaptic inputs. C. They deliver synaptic input on other neurons. D. They make synapses on effector organs in the PNS. E. They can transmit information between afferent neurons and efferent neurons.

d

48. Which is NOT true about cofactors involved in enzymatic reactions? A. They may alter the conformation of the enzyme. B. They may be a metal such as iron. C. They may be a substrate in a catalyzed reaction. D. There must be equal quantities of enzyme and cofactor molecules for a reaction to proceed. E. Only a small number of cofactor molecules may be needed to maintain enzyme activity.

d

50. Which of the following is true concerning the rate-limiting step in a metabolic pathway? A. It is likely to be the slowest reaction in the pathway, and it is always the last step in a metabolic pathway. B. It is likely to be the fastest reaction in the pathway, and it is always the last step in a metabolic pathway. C. It is always the first step in the pathway, and may be subject to end-product inhibition. D. It is likely to be the slowest reaction in the pathway and it may be subject to end-product inhibition. E. It is likely to be the fastest reaction in the pathway and it may be subject to end-product inhibition.

d

57. What are the products of glycolysis under anaerobic conditions? A. 2 molecules of NADH and 2 molecules of water B. 2 molecules of pyruvate, 2 molecules of lactate, and 2 ATP C. 2 molecules of pyruvate and 36 molecules of ATP D. 2 molecules of ATP, 2 molecules of water, and 2 molecules of lactate E. 2 molecules of pyruvate, 2 molecules of water, and 2 NADH

d

6. The cell type that is specialized to communicate with other cells and control their activities is A. Epithelial cells B. Muscle cells C. Connective tissue cells D. Nerve cells

d

60. Which is one of the major inhibitory neurotransmittors in the CNS? A. Glutamate B. Dopamine C. Norepinephrine D. Gamma-aminobutyric acid (GABA) E. Beta-endorphin

d

63. The portion of the peripheral nervous system that is composed of nerve fibers that innervate skeletal muscle is called the: A. afferent nervous system. B. sympathetic nervous system. C. parasympathetic nervous system. D. somatic motor nervous system. E. autonomic nervous system.

d

64. Before an amino acid can be broken down for energy: A. the side chain must be removed. B. the carboxyl group must be removed. C. it must be converted to NAD+. D. the amino group must be removed. E. it must be phosphorylated.

d

69. Which best describes the reticular formation of the brain? A. It is mainly involved in motor coordination and balance. B. It is the master endocrine gland of the brain. C. It is primarily responsible for visual perception. D. It integrates information from all regions of the CNS, and incorporates the mechanisms that regulate sleep and wakefulness. E. It is the primary synaptic relay station for sensory information entering the CNS.

d

7. In general, polar molecules diffuse more rapidly through the lipid bilayer part of cell membranes than do nonpolar molecules. This statement is: A. true. B. false, because polar molecules diffuse through less rapidly because the bilayer is polar throughout its width. C. false, because polar molecules diffuse through less rapidly because the bilayer is nonpolar throughout its width. D. false, because nonpolar molecules diffuse through more rapidly because much of the bilayer is nonpolar. E. false, because the rate of diffusion of nonpolar and polar molecules through the bilayer is essentially the same as long as the molecules are the same size.

d

74. Postganglionic neuron cell bodies of the autonomic nervous system have which category of neurotransmitter receptor? A. Adrenergic receptor B. Serotonin receptors C. Muscarinic acetylcholine receptors D. Nicotinic acetylcholine receptors E. Dopamine receptors

d

8. Compartments A and B are separated by a membrane that is permeable to K+ but not to Na+ or Cl-. At time zero, a solution of KCl is poured into compartment A and an equally concentrated solution of NaCl is poured into compartment B. Which would be true once equilibrium is reached? A. The concentration of Na+ in A will be higher than it was at time zero. B. Diffusion of K+ from A to B will be greater than the diffusion of K+ from B to A. C. There will be a potential difference across the membrane, with side B negative relative to side A. D. The electrical and diffusion potentials for K+ will be equal in magnitude and opposite in direction. E. The concentration of Cl- will be higher in B than it was at time zero.

d

8. Epinephrine activates the cyclic AMP pathway in liver cells. In this example, epinephrine is a ____________ and cAMP is a _____________. A. ligand, receptor B. first messenger, hydrophobic hormone C. second messenger, ion channel D. first messenger, second messenger E. enzyme, second messenger

d

8. Which best describes the extracellular matrix? A. It is found just inside the cell membrane in all tissues, it sends branching collagen fibers between cells to connect them, and it transmits chemical information from the interior of one cell to the interior of adjacent cells. B. It is a tissue having more than the four general cell types, it transports proteins and polysaccharides between body compartments, and it is the route by which chemical signals like hormones reach all parts of the body. C. It covers the body's surface, it contains connective and muscle tissue, and it helps generate movement. D. It surrounds cells, it contains proteins, polysaccharides and minerals, it provides a scaffold for cell attachment, and it transmits chemical messengers to cells.

d

1. Which is a major function of the plasma membrane? A. storing calcium ions B. storing organic chemicals for metabolism C. providing genetic information D. generating ATP E. regulating the passage of molecules into and out of the cell

e

1. Which of the following is/are functions of the human nervous system? A. receiving, storing, and processing information on the internal and external environments B. bringing about changes in physiology and/or behavior to ensure optimal functions of homeostatic mechanisms C. secretion of hormones D. coordination of movement E. All of the choices are correct.

e

11. Which organelles would you expect to be especially numerous in cells that utilize oxygen to generate a great deal of energy in the form of ATP? A. peroxisomes B. endosomes C. lysosomes D. ribosomes E. mitochondria

e

12. Which of the following is a feature that distinguishes active transport from facilitated diffusion? A. saturation of transport rate B. requirement for a carrier molecule C. carrier molecules have specificity D. presence of a transport maximum E. requirement for metabolic energy

e

12. Which of these are functions of the Golgi apparatus? A. modifying proteins synthesized on free ribosomes and storing calcium ions B. digestion of engulfed bacteria and using oxygen to generate ATP C. synthesizing lipids and intracellular storage of calcium ions D. modifying proteins synthesized on free ribosomes and allowing the distribution of modified proteins throughout the cell in vesicles E. modifying proteins synthesized on ribosomes associated with granular endoplasmic reticulum and allowing the distribution of modified proteins throughout the cell in vesicles

e

14. Which of components of a general reflex arc are listed in the order information typically flows through them following a stimulus? A. effector, afferent pathway, integrating center, efferent pathway, receptor B. effector, efferent pathway, integrating center, afferent pathway, receptor C. integrating center, receptor, afferent pathway, efferent pathway, effector D. receptor, efferent pathway, integrating center, afferent pathway, effector E. receptor, afferent pathway, integrating center, efferent pathway, effector

e

15. By what transport mechanism does glucose enter most cells? A. diffusion through the lipid bilayer B. primary active transport C. secondary active transport D. diffusion through a protein channel E. facilitated diffusion

e

16. Which is FALSE about the equilibrium potential of a given ion across a membrane? A. It is a function of the concentration of that ion on both sides of the membrane. B. It is the potential at which there is no net movement of that ion across the membrane. C. It is the potential difference across the membrane at which an electric force favoring movement of the ion in one direction is equal in magnitude and opposite in direction to the diffusion force provided by the concentration difference of the ion across the membrane. D. A permeable ion will move in the direction that will tend to bring the membrane potential toward that ion's equilibrium potential. E. An anion that is in higher concentration inside the cell than outside the cell will have a negative eqilibrium potential.

e

17. Which of the following statements about the Na+-K+ ATPase pump is false? A. It transports Na+ out of cells and K+ into cells. B. It binds to, and hydrolyzes, ATP. C. It is constantly active in all cells. D. Its activity requires the expenditure of metabolic energy. E. It transports Na+ and K+ in a 1:1 ratio.

e

18. Shivering in response to a cold draft is an example of A. A homeostatic mechanism B. Negative feedback C. A physiological reflex D. Thermoregulation E. All of the choices are correct

e

19. What is a genome? A. a cluster of genes that are all regulated in the same manner B. a measure of the genetic variability in a population C. a region of DNA that codes for a single protein D. all of the protein in a cell at a given point in time E. the total genetic information in a typical cell of an organism

e

19. Which is NOT an example of a graded potential? A. a receptor potential in a sensory receptor cell B. a depolarizing excitatory postsynaptic potential (EPSP) C. a hyperpolarizing inhibitory postsynaptic potential (IPSP) D. a depolarizing pacemaker potential E. a depolarizing action potential

e

2. Specificity is an important characteristic of intercellular communication; which of these best explains why it occurs? A. because all cells have very similar compositions of their phospholipid membranes B. because protein receptors are only located on the surface of target cells C. because all cells have the same DNA, so any cell can express protein receptors for a specific chemical D. because chemical messengers are all proteins E. because protein receptors for chemical messengers are only expressed in specific target cells

e

20. Amplification of a second messenger cascade can take place at which level of a signal cascade? A. One activated receptor can activate numerous G-proteins. B. One activated G-protein can activate numerous effector enzymes. C. One active effector enzyme can catalyze numerous reactions. D. One activated protein kinase can allosterically modulate numerous proteins. E. All of the choices are correct.

e

20. What is the best description of the efferent pathway of a reflex arc? A. signals from the integrating center to receptors B. the route by which receptors send signals to effectors C. signaling pathway for receptors to influence the integrating center D. the route by which effector organs send signals to receptors E. the route by which signals from an integrating center reach effector organs

e

21. What is a role of calcium ions in the second messenger cascade involving phospholipase C, diacylglycerol, and inositol trisphosphate? A. It splits and activates G-protein subunits. B. It binds to the endoplasmic reticulum and causes the release of inositol trisphosphate. C. It phosphorylates cell proteins. D. It is the first messenger that binds to the integral membrane protein receptor. E. Along with diacylglycerol, it activates protein kinase C.

e

25. What is a hormone? A. a chemical released from a nerve cell that affects nearby cells across a synapse B. a chemical released from an endocrine gland that affects target cells without entering the bloodstream C. a chemical found in the blood that catalyzes the destruction of ingested toxins and foreign substances D. a chemical excreted from sweat gland that signals other individuals about the physiological status of the body E. a chemical regulator secreted from an endocrine gland that travels through the bloodstream to affect target cells

e

26. Which is NOT a function of the Golgi apparatus? A. sorting of proteins destined for various locations in the cell B. modifying proteins destined for secretion C. packaging enzymes destined for lysosomes D. addition of carbohydrate groups to proteins E. transcribing a signal sequence onto proteins destined for secretion

e

26. Which is most directly responsible for the falling (repolarizing) phase of the action potential? A. Voltage-gated Na+ channels are opened. B. The Na+, K+ pump restores the ions to their original locations inside and outside of the cell. C. The permeability to Na+ increases greatly. D. ATPase destroys the energy supply that was maintaining the action potential at its peak. E. The permeability to K+ increases greatly while that to Na+ decreases.

e

29. Which of the following statements about the refractory period of a membrane is true? A. The absolute refractory period refers to the period of time during which another action potential cannot be initiated in that part of the membrane that is undergoing an action potential, no matter how great the strength of the stimulus. B. The relative refractory period refers to the period of time during which another action potential can be initiated in that part of the membrane that has just undergone an action potential if a stronger than normal stimulus is applied. C. The refractory period prevents the action potential from spreading back over the part of the membrane that just underwent an action potential. D. The refractory period places an upper limit on the frequency with which a nerve cell can conduct action potentials. E. All of the above choices are correct.

e

3. Which of the following are ways in which binding of an intercellular chemical messenger with a cell's receptor can bring about a cellular response? A. opening or closing of specific ionic channels in the plasma membrane B. activation of an intracellular second-messenger system C. promoting or inhibiting the transcription of genes that code for the synthesis of cellular proteins D. activating or inhibiting intracellular enzymes E. All of the choices are correct.

e

30. A solution containing proteins of a particular type is exposed to the same concentrations of ligands X and Y, but the percent saturation of molecule X is greater than the percent saturation of molecule Y. Which is most likely to be a true statement? A. The binding sites for ligand X are 100% saturated. B. The binding sites for ligand X have greater specificity than the binding site for ligand Y. C. Ligand X must be a competitive inhibitor of ligand Y. D. Both ligands must bind to the same binding site on the protein. E. The protein has a higher affinity for ligand X than for ligand Y.

e

31. Circadian rhythms are biological rhythms with what main characteristic? A. They are cyclical, like the 28-day female menstrual cycle. B. They are cyclical, like the rhythmic beating of the heart. C. They are voluntary rhythms, like the time you decide to eat lunch each day. D. They cease to occur when a person is in a dark environment. E. They repeat approximately every 24 hours, like daily spikes in hormone secretion.

e

33. What process lowers blood plasma levels of cholesterol-containing lipoproteins? A. second messenger-activated, sterol transgenesis B. hydrophobic phagocytosis C. simple, transcellular membrane diffusion D. Na+ gradient-dependent, secondary active transport E. clathrin-dependent, receptor-mediated endocytosis

e

36. An action potential does not re-stimulate the adjacent membrane that was previously depolarized because A. stimulation is inhibited by the myelin sheath. B. it is impossible for an action potential to be propagated along an axon toward the nerve cell body. C. the resting membrane potential of the axon is too positive. D. the resting membrane potential of the axon is too negative. E. that area of the membrane is in the absolutely refractory period.

e

37. Which is NOT true about protein kinases? A. They add phosphate groups to proteins. B. They produce effects similar to allosteric modulation. C. They perform the opposite function as phosphatases. D. They use ATP to carry out their reactions. E. They remove phosphate groups from proteins.

e

38. The term "metabolism:" A. is synonymous with the term "catabolism." B. is synonymous with the term "anabolism." C. refers to any chemical reaction that involves the production of energy. D. is defined as the covalent modification and activation of a protein with binding sites for organic molecules. E. refers to the synthesis and breakdown of organic molecules involved in cell structure and function.

e

39. The probability of a reaction occurring is increased A. by decreasing reactant concentrations. B. by increasing the activation energy. C. by increasing the concentration of any of the products. D. if the reaction uses a great deal of energy. E. if the concentration of one of the reactants increases.

e

4. What feature of phospholipids allows them to interact with aqueous cytosol and extracellular fluid, while still presenting a barrier to hydrophilic substances? A. They are polar molecules. B. They are nonpolar molecules. C. They are lipids. D. They are extremely rigid molecules. E. They are amphipathic molecules.

e

4. Which is FALSE about neurons? A. A given neuron can be either a presynaptic neuron or a postsynaptic neuron. B. An individual neuron can receive information from multiple other neurons. C. An individual neuron can transmit information to multiple other neurons. D. A neuron can simultaneously release more than one type of neurotransmitter. E. A neuron receives information on its axons and delivers it to other neurons through its dendrites.

e

43. Which of the following statements about EPSPs is false? A. They are produced by the opening of chemically-gated sodium channels. B. They transmit signals over relatively short distances. C. They depolarize postsynaptic cell membranes. D. They are able to summate. E. They are always the same amplitude.

e

53. The site where most of the ATP is generated in a cell is the A. nucleus. B. plasma membrane. C. endoplasmic reticulum. D. Golgi apparatus. E. mitochondria.

e

54. Acetylcholine is the main neurotransmitter released by: A. preganglionic sympathetic neurons and cells of the adrenal medulla. B. cells of the adrenal medulla and postganglionic parasympathetic neurons. C. motor neurons and postganglionic sympathetic neurons. D. preganglionic sympathetic neurons and postganglionic sympathetic neurons. E. preganglionic sympathetic neurons and motor neurons.

e

55. Which of the following statements about norepinephrine is FALSE? A. Norepinephrine is a neurotransmitter. B. Norepinephrine binds to adrenergic receptors. C. Norepinephrine is a catecholamine. D. Dopamine is a precursor to norepinephrine. E. Epinephrine is a precursor to norepinephrine.

e

6. What are desmosomes? A. low-resistance channel-like passages that let ions travel between the interiors of adjacent cells B. cell organelles that contain enzymes for digesting cellular debris and foreign microbes C. cellular organelles that transcribe RNA into DNA D. membrane-bound vesicles that pinch off from the plasma membrane and enter the cell E. dense plaques of proteins that maintain firm attachments between adjacent cells

e

61. Which is NOT true about fatty acid synthesis? A. takes place in the cytosol of cells B. results in molecules with an even number of carbon atoms only C. requires more energy than is produced by the catabolism of the same fatty acid D. begins with a molecule of actetyl coenzyme A E. The enzymes that catalyze it are in the same location as those that mediate fatty acid catabolism.

e

62. The central nervous system includes the: A. afferent nerves and spinal cord. B. efferent nerves and spinal cord. C. autonomic nervous system and the brain. D. brain stem and the autonomic nervous system. E. brain and spinal cord.

e

64. The region of the brain that is the most important control area for homeostatic regulation of the internal environment is: A. the thalamus. B. the hippocampus. C. the cerebrum. D. the cerebellum. E. the hypothalamus.

e

9. Which best describes functions of the agranular (smooth) endoplasmic reticulum? A. transcription of DNA into RNA B. generation of ATP C. digestion of engulfed bacteria and cellular debris D. synthesis and packaging of proteins for secretion from the cell or export to other organelles E. synthesis of lipids and intracellular storage of calcium ions

e


Ensembles d'études connexes

I.Plot Quesrions for The Tell -Tale Heart and Comprehension and Style Activities

View Set

Chpt 15 Antiparkinson Drugs Pharmacology

View Set

Psych 232 Chapter 6 and 7 Study Guide

View Set

Chapter 6: Markets and Social Security

View Set

Chapter 5 - Small Business, Entrepreneurship and Franchising

View Set